SlideShare a Scribd company logo
1 of 43
Download to read offline
50 star ccs cases - USMLE Forums                                                       http://www.usmle-forums.com/usmle-step-3-recommended-thre...



                                            USMLE Forums
                                               Your Reliable
                                          USMLE Online Community
                                             28,025 Members
                                              98,912 Posts


                   Home              USMLE Articles   USMLE News         USMLE Polls        USMLE Books        USMLE Mobile       Monthly Contest

                                                                                                               User Name User Name      Remember Me?
               USMLE Forums > USMLE Step 3 Forum > USMLE Step 3 Recommended Threads
                50 star ccs cases                                                                              Password                Log in

                    Register                FAQs               Community                           Today's Posts                     Search

            Hi, Unregistered

             You have limited access as a guest. Join our community to have full access. Registration is fast, simple and absolutely free.

                        Get your Free USMLE Consult™ Step 3 Question bank subscription when you post more in USMLE Step 3 Forum!
                                                              Click Here for more details.




             USMLE Step 3 Recommended Threads Highly Recommended and Useful USMLE Step 3 Forum Threads.




                                                                                             Thread Tools    Search this Thread      Display Modes

              09-23-2009                                                                                                                           #1

                                                                                                                             Steps History: 1+CK+CS
                                                                                                                             Posts: 3
             navz                                                                                                            Threads: 1
             USMLE Forums Newbie                                                                                             Thanked 10 Times in 1 Post
                                                                                                                             Reputation: 20

                50 star ccs cases


             found this on another forum so thought of sharing here



             . femur neck#--ER

             2. Advanced maternal age

             3. Snake bite

             4. HEPATIC ENCEPHALOPATHY

             5. Post-op atelectasis.

             6. Septic Abortion-

             7. Incomplete abortion with heavy bleeding and acute PID.

             8. Infected peripheral IV access

             9. atrophic vaginitis

             10. polycystic ovarian syndrome

             11. gono

             12. AGN

             13. BPH

             14. acute psychotic disorder

             15. spontaneous abortion




1 of 43                                                                                                                                          4/16/12 11:27 AM
50 star ccs cases - USMLE Forums                                  http://www.usmle-forums.com/usmle-step-3-recommended-thre...



             16. malrotation

             17. child with rash

             18. obesity in adolescent

             19. CTS

             20. teenager comes for a pre-employment

             21. JRA

             22. kawasaki disease

             23. Osteomyelitis

             24. Hypoglycemia

             25. Dilated cardiomyopathy

             26. Down's Syndrome neonate

             27. Preeclampsia

             28. MM

             29. Hypercalcemia-office

             30. Hypercalcemia-ER

             31. MS

             32. HELLP

             33. Endometrial Hyperplasia without Atypia

             34. Gastric cancer

             35. TURP-HYPONATREMIA

             36. testicular pain-acute unilateral, child—ER

             37. hypertensive crisis with SAH

             38. Hypertension-secondary

             39. fever unknown origin – adult

             40. septic shock

             41. Alcohol withdrawal

             42. retained placenta

             43. Chronic renal failure

             44. Acute renal failure

             45. RAPE

             46. HUS

             47. New onset DM-42 yr old c polydipsia & polyuria

             48. Fever Unknown origin-child

             49. Cx ca




2 of 43                                                                                                      4/16/12 11:27 AM
50 star ccs cases - USMLE Forums                        http://www.usmle-forums.com/usmle-step-3-recommended-thre...


             50. Failure to Thrive



             1-femur neck#--ER

             Hx- 55 female post meno
             Exter. Rot. & shorten Right LL
             VS- stable
             P/E
             general
             skin
             heent look for hematoma
             heart
             lung
             abd
             cns
             extr

             Orders

             IVA
             oxy sat
             Pulse oxy
             cardiac monitor
             bp monitor
             EKG
             nss
             cbc
             bmp
             ua

             pt
             ptt
             inr
             fobt
             blood type,crossmatch
             Ca,phoph,pth,mg
             x ray –chest,hip(ap/lat) ,knee
             morphin-pain
             consent form
             orthopedic consult ; reason hip fracture

             admit to wd
             interval+rest ex
             cefazolin IV on time
             npo
             bed rest ,bathroom
             urine output
             foley
             ranitidine
             cbc/d
             bmp/d
             h&H
             ptt-4h
             heparin sq
             pneumatic

             surgery done-
             post-sx
             morphi
             ambulate early
             calcium
             vit d
             if shows patient improves,

             discharged and f.u after 2wks

             Counseling
             mobilization




3 of 43                                                                                            4/16/12 11:27 AM
50 star ccs cases - USMLE Forums                                                       http://www.usmle-forums.com/usmle-step-3-recommended-thre...


             exercise
             alendronate
             medicine comp
             stop smoking
             limit alcohol
             seat belt
             calcium rich diet
             Dexa scan
             colonoscopy
             lipid
             mammography

             ---------------------------------

             2-Advanced maternal age
             office:
             PE--complete.
             urine HCG (+ve)

             CBC
             BMP
             UA
             urine culture
             LFTs
             lipid profile
             blood type & cross
             type and Rh
             pt/ptt
             EKG
             pap smare
             USG trans vaginal
             mammography
             gonorrhea
             chlymydia
             RPR
             Hep B antibody
             HIV ELISA
             toxo
             rubella titer

             Educate patient pregnancy
             pregnant mother counseling
             no smoking, safe sex, seat belt, safety plan
             diet (high calory,High protein)
             iron oral
             folic acid oral
             multi vitamins-prenatal

             follow up after 1 month in 1st trimester
             2nd visit
             full physical
             vital(BP check)
             Weight
             UA
             Fetal heart monitoring
             fundal height



             do triple screen --serum HCG, AFP, estradiol [MS AFP], if high amniocentesis
             Triple marker screen-TRIPLE TEST
             Valproic acid level

             send her home..

             counsel about the vit and folic and RATED SEX

             further diagnostic plan
             continue Valproic acid
             Genetic counseling




4 of 43                                                                                                                           4/16/12 11:27 AM
50 star ccs cases - USMLE Forums                                                        http://www.usmle-forums.com/usmle-step-3-recommended-thre...


             Genetic Sonogram
             Amniocentesis
             Karotyping of the fetal amniocytes....cells found in the amniotic fluid

             ---------------------------------
             3-Snake bite

             ER Location
             25 y young guy – 30min after hx of snake bite.
             Haven’t brought snake,

             order-
             vitals/hr
             IV Access
             NS
             Pulse oxi
             O2
             Abg
             Cont BP moniter
             Cardiac moniter
             cbc
             bmp
             ua
             pt
             ptt
             Bleeding time
             ECG
             CXR
             Blood type and cross match.
             Foley
             Urine output
             NPO

             PE-(complete)
             shows local cellulitis, noticed 2 fang sites on his ankle Neuro xam shows drowsiness
             order-
             Polyvalent snake antivenom -SNAKE
             Shift to ICU
             interval hx+rest Ex
             Bedrest
             Vitalsq2h
             Pulse Oximetry q2h
             Neuro exam q2h
             Monitor Bleeding time, PT, PTT, Platelet counts (Can develop DIC)
             H&H-6h
             Ranitidine
             Inj TT
             Ampicillin/Cloxacillin IV
             ABG q8h
             cbc/24
             Bmp/24

             If Neuroparalyis symptoms---- (Atropine + Neostigmine IV)
             If devloping resp failure---- intubate and mech ventilation

             D/C ASV when Bleeding time/PT/PTT parameters normal and neuro symptoms subside,
             D/c IV antibiotics; make oral
             When pt ok—to wd Later send home
             counselling
             -----------------
             4-HEPATIC ENCEPHALOPATHY

             pt presented with altered state of mind... had h/o of cirrhosis of liver already. was given oxycodone my dentist following which he
             devleloped symptoms..HEP C cirrhosis

             LOOK FOR-alkalosisi,low k,SBP,HIGH PROT diet,…..

             ER
             routine (IV access, pulse oximetry, cont bp, cont cardiac moniter)




5 of 43                                                                                                                                            4/16/12 11:27 AM
50 star ccs cases - USMLE Forums                                                          http://www.usmle-forums.com/usmle-step-3-recommended-thre...


             fingers stick glu
             Thiamine inj
             50% dxt

             NGT suction (to look for variceal bleeding)
             EKG
             CXR
             CBC, BMP, S.NH3
             NPO
             NSS

             GPE( signs of cirrhosis, confused patient, asterixis)

             LFT, PT, PTT, urine r/e, toxicology, blood alcohol level ,Mg
             URINE culture
             BLOOD culture
             (pt marginally raised, lft deranged as for cirrhosis serum NH3 sky high)

             admit ICU

             urine output, bed rest, continue thiamine and dextrose normal saline drip,

             propranolol to control portal HTN./ spironolactone

             PARACENTESIS-SEND FLUID

             LEVOFLOXACIN----IF PARACENT+VE{>250CELL) CHANGE TO CEFTRIAXONE

             .Lactulose oral
             [ampicillin po or neosporin po/ neomycin]
             enemas to evacuate stool

             Moniter PT/PTT/ cbc/bmp/ammo daily .

             Pt improves –ADD diet salt restriction

             .shift to wards in 24 hrs or when better

             Case usually ends after 6 hrs of pt getting better.

             rehab

             5-Post-op atelectasis.

             [after 36 hr of Surgery]...
             DD's were:
             Pneumothorax
             PE
             Pneumonia

             CXR--it showed Atelactasis

             It is one of the first cause of Fever, High WBC and shortness of breath...

             So..

             I did..blood culture
             Removed the Foley Catheter and put a new one
             UA culture
             And Started..
             IV Antibiotics
             Acetaminophen Per Rectal

             And Incentive Spirometry...

             Patient become stable...I transferred him to Inpatient Unit..

             ----------------
             6-Septic Abortion-




6 of 43                                                                                                                              4/16/12 11:27 AM
50 star ccs cases - USMLE Forums                                                         http://www.usmle-forums.com/usmle-step-3-recommended-thre...



             Do cultures
             IV Antibiotics
             Call OBGYN
             When patient is accepeted by OBGYN for D&E

             Just do medical Management in ICU

             7-Incomplete abortion with heavy bleeding and acute PID.

             Do cultures
             IV Antibiotics
             Call OBGYN
             When patient is accepeted by OBGYN for D&E

             Just do medical Management in ICU
             --------------

             8-Infected peripheral IV access

             iva (if central line, dc cental line and new central line)
             oxy
             vitals q1h (qday when stable temp)
             cardiac monitor (risk of septic shock)
             fingerstick stat
             b-hcg
             cbc stat
             bmp stat

             focused pe

             pt/ptt stat
             ua/uc+s
             blood cx
             cxr
             esr
             crp
             xray site
             doppler arm
             remove iv line
             cath tip for c+s, gm stain, fungal cx
             clinda + zosyn (if admitted >48 hrs)
             tylenol
             ekg
             2 d echo

             full pe
             elevate arm
             bed rest
             iv nss
             iv heparin if signs of cord-like thickening/bluish discoloration
             (new iv access already done in beginning to give empiric abx)

             when cx back:
             iv naf for 2-3 days (until improvement)
             dc zosyn and clinda
             in this case do not worry abt dc'ing patient or po meds and patient has iv line for a reason.

             5 min:
             repeat cbc, chem in 3-4 days
             counsel
             screen
             ----------------
             9-atrophic vaginitis

             62 yo
             vaginal itching
             clear discharge
             painful intercourse




7 of 43                                                                                                                             4/16/12 11:27 AM
50 star ccs cases - USMLE Forums                                                       http://www.usmle-forums.com/usmle-step-3-recommended-thre...



             vulvar erythema
             mucus bleed during exam

             dd

             atrophic vag
             bacterial vagi
             candidial vagi
             vulvar ca
             cervicitis
             tricho

             office work up

             cbc cmp lipid profile as a health maintainance exam-safer to do bu no credit

             wet prep
             trich
             gono
             chly
             pap
             may need emb colpo etc if finding in cervix or pap

             if pap positive story goes further
             otherwise
             cou

             vaginal gelly for lubrication
             local hrt
             estrogen cream

             follow up as needed

             -----------------
             10-polycystic ovarian syndrome

             21 yo f
             beard
             excessive hair
             weight gain
             menstrual irregularity
             darkening axillry thickened skin
             normal vitals

             dd

             polycystic ovarian syndrome
             congenital late onset adrenal hyperplasia
             adrenal tumour
             drug effects like minoxidil phenytoin
             ovarian neoplasm
             cushing syndrome
             idiopathic hirsutism

             cbc lft

             bmp

             endocrine--dhea

             lh fsh ratio

             prolactin

             tsh

             dhea




8 of 43                                                                                                                           4/16/12 11:27 AM
50 star ccs cases - USMLE Forums                     http://www.usmle-forums.com/usmle-step-3-recommended-thre...


             testerone

             blood glucose

             insulin level

             serum 17 hydroxyprogesterone

             us pelvis

             result - testerone increse

             lh fsh ratio increase
             insulin fasting glucose ratio increse

             urine pregnancy test -do it anyway --
             rx
             ocp
             exercise
             metformin
             spirolactone
             smoking cesation

             fu 6 month

             ----------------
             11-gono-male

             21 m
             unprotected sex
             urethral discharge fever

             sickness
             burning sensation during urination
             o/e

             urethral discharge
             red urethra
             suprapubic tender

             d/d

             -acute cystitis
             epidymitis
             forign body
             nephrolithiasis
             orchitis
             prostitis
             pyelonephritis
             reiter's syndrome

             urethritis
             --gono
             chlymydia

             office w/u
             ua culture urethral gram stain

             urethral discharge for gono
             chlymydia
             vdrl
             cbc

             finding -- gram stain gram neg
             culture awited

             rx
             azithromycin 1 gram stat




9 of 43                                                                                         4/16/12 11:27 AM
50 star ccs cases - USMLE Forums                        http://www.usmle-forums.com/usmle-step-3-recommended-thre...


             ceftrixone 250 mg stat

             coun safe sex practice
             smoking
             alcohol
             safe driving
             drug

             culture --
             fu 4 week
             pt coun

             treat patner

             -------------
             12-AGN

             10 m
             tea urine
             priorbit edema
             had fever with hx of sore thrat 3 wk bak

             bp 140/85
             ankle edema

             dx
             -cryoglobunemia
             iga nephropathy
             membranoprliferative gn
             post streo gn

             er work
             cbc chem 8
             ua
             no need of cs he does not have fever

             24 hour urine protein
             aso titer
             complement -low

             ua--proteinuria
             wbc cast
             rbc cast

             rx

             lasix
             captropril
             penicillin

             office work up
             us renal
             throat culture
             office rx

             furosemide
             captropril
             nephrology consult -

             fu 3 week
             family couns
             dietary consult
             low sodium diet
             fluid restriction
             seat belt

             ----------
             13-BPH
             70m sono need of pregnancy test?




10 of 43                                                                                           4/16/12 11:27 AM
50 star ccs cases - USMLE Forums                                                          http://www.usmle-forums.com/usmle-step-3-recommended-thre...



             night urin
             frequency urgency hesitency
             terminal dribbling
             double micturation
             weak stream
             sensation of incomplete evacuation

             vita wnl
             prostate normal but enlarged

             office
             cbc
             bmp-urea creatinine normakl
             ua
             cs

             us prostate
             psa
             esr
             residual urinary volume

             rx
             finesteride
             prazocin which is a selecting short acting alpha blocker



             second visit

             urology consult
             urodynamic study



             fu six month
             for dre and psa

             dietary consult
             seat belt smoking
             alcohol
             patient counseling
             -----------

             14-acute psychotic disorder

             dd
             mania
             bipolar 1
             stress
             malingering
             panic
             scizophrenia
             drug
             delirium

             vital s -wnl

             so pe

             meds-
             olazapine
             valproic acid
             we should give a antipsychotic and mood sabliser- lithium or valproic acid

             order-cbc bmp no need of lft pt ptt order tsh uds no need of ua no need of ekg xray cardiac enzymes

             do psychotherpy
             psychiatry consult

             coun med compliance




11 of 43                                                                                                                             4/16/12 11:27 AM
50 star ccs cases - USMLE Forums                               http://www.usmle-forums.com/usmle-step-3-recommended-thre...


             suicide contract
             regular exercise
             patient education

             send home
             appointment - one week

             agin exam

             if not allright -vdrl hiv sle

             ect can be given

             monitor cbc with antipsychotic
             -------------
             15-spontaneous abortion

             27 yo f
             lmp 6 week ago
             lower abdominal crmp
             vag bleed

             cervix - open
             blood in vault
             vitals tachy bp wnl
             dd

             ectopic
             abortion
             polyp
             cancer inflammation or cervicitis

             normal menstration with dysmenorrhoea

             er work up
             cbc
             pregnancy test
             qualitative
             then quantiatative
             us
             blood group rh

             iv saline
             no cervix tenderness - no pap gono cz now

             hb -9 no bllod transfusion now pt is stable

             us -- fetus dead - fetal pole uterine pregnancy

             gyn consult for d and c
             d and c

             admit to ward

             iv saline
             pneumatic compression
             methylprednisolone
             doxy
             cbc folow up

             grief counselling
             counsel pt rebirth control

             follow up 3 week

             -------
             16-malrotation VOLVULUS

             1 dy old m bilious vomi




12 of 43                                                                                                  4/16/12 11:27 AM
50 star ccs cases - USMLE Forums                                        http://www.usmle-forums.com/usmle-step-3-recommended-thre...


             poor feed
             lethargy
             rectal bleed

             oe-

             distension
             170 pulse
             89 sat

             dd

             duodenal atresia
             intestinal atresia
             malrotation with volvulus
             meconium plug

             necrotising enterocolitis

             will do gi series to r/o duodenal
             will do plain xray
             will r/0 infections

             transfer to er
             iv aceess
             iv normal saline

             o2
             abg
             cbc
             bmp
             lft
             abdominal xray
             cxr
             BLOOD C/S if fever
             abg-metabolic acidosis- means something in the intestine

             cbc leucocytosis-
             axr-airless rectum
             large gastric bubble- means some obstruction



             rx as intestinal obst
             rx--NPO
             ng tube suction

             iv bicarb if ph less than 7

             pediatric surgery consult

             ward -

             upper gi sries -
             barium enema
             ng tube suction



             upper gi -- bird beak
             corkscrew proximal jejunum

             barium enema cecum in RUQ



             rx ng tube suction
             iv normal saline bmp



             fu 48 hours




13 of 43                                                                                                           4/16/12 11:27 AM
50 star ccs cases - USMLE Forums                                                        http://www.usmle-forums.com/usmle-step-3-recommended-thre...


             family counselling

             ----------------------

             17-child with rash

             ruleout drug reactin??
             rule out lyme if suspected

             Office W/U
             Complete PE
             CBC, stat
             BMP, stat
             Pulse oximetry
             ESR, routine
             blood culture
             UA, urine culture
             CXR, stat
             EKG, 12 lead, stat
             Neck x ray, stat
             culture of scraping from rash
             No aspirin

             send the patient home and F/U in 4 days

             -------------
             18-obesity in adolescent

             Complete PE

             Measure Height
             Measure Weight
             Calculate BMI -- you have to do this manually...not in the CCS software

             cbc
             BMP
             LFT
             Fasting Plasma Glucose
             Fasting Lipid Profile
             Serum TSH
             UA
             24 hour urinary cortisol

             If the age of the PT is 2-7 years old and BMI with 95 percentile......or more without complication, the goal should generally be
             maintenance of baseline weight,

             For children 2–7 years old with BMI at the 95th percentile or above and secondary complications, weight loss is indicated

             For children older than 7 years with BMI between the 85th and 95th percentile, without complications, weight maintenance is an
             appropriate goal.

             I guess this patient's BMI >95th percentile

             so,

             Weight Loss diet
             Counsel Patient for Exercise Program
             Counsel Patient
             Counsel Family



             Follow-up visit in 4 months

             If no change...

             Sibutramine or Orlistat, po
             Follow up in 6 weeks
             If morbid obesity, BMI more than 45




14 of 43                                                                                                                                        4/16/12 11:27 AM
50 star ccs cases - USMLE Forums                                                       http://www.usmle-forums.com/usmle-step-3-recommended-thre...


             Consider Bariatric surgery
             -------------
             19-CTS

             pe

             xray

             cbc
             esr
             crp
             tsh
             ra
             ana
             ca
             magnesium
             bmp
             lipid
             hcg



             fu 3 days
             night splint-SPLINT EXTREMITY
             nerve conduction study
             nsaid

             usual counselling

             another visit

             carpal tunnel confirmed
             another cou

             3 month

             not better
             ortho consult for surg




             20-teenager comes for a pre-employment

             she is 5ft 2 inches and weighs 180 lbs. Bp 155/90

             pt eats fast food..
             it is all about weight loss....



             bmp show no Potasium drop
             abdominal exam ...no striae.... PE does not point towards high cortisol
             for her BP

             do urinanalysis
             chest x ray
             ekg
             bmp

             for her bmi
             lipid profile fasting
             FBS
             TSH
             cbc

             follow up one week

             bp in both arm
             counselling life style -
             low sodium low fat low cholesterol




15 of 43                                                                                                                          4/16/12 11:27 AM
50 star ccs cases - USMLE Forums                                                         http://www.usmle-forums.com/usmle-step-3-recommended-thre...


             weight mange
             oreder calorie count and exercise
             folow up three month

             better

             not better add hctz

             it is not secondary hypertension bmp normal no cushing no coarctation

             ----------------
             21-JRA

             7 yr old girl with fever, rash and polyarthritis
             PE: Complete

             Order:
             cbc ( stat )
             bmp or may be just BUN and Creatinine (stat)
             ESR
             ANA
             RF
             UA
             LFTs
             Xrays of the joints involved
             PT/PTT ( if planning to do arthrocenthesis )
             athrocenthesis can be done as well....

             CULTURE-URINE/BLOOD/THROAT
             Tylenol
             CXR
             12 Lead EKG
             Echo
             d-dimers and fibrinogen ( i do not know the indication ..maybe coz it's a vasculitis)

             will pretend the child does not have a high fever...so send him home. see in next 4-5 days with the lab results..

             Results:
             Cxr of joint without erosive changes...( so no methorexate)
             ANA positive
             such pts get eye involvement, thus eye exam q 3mths
             RF positive
             BUN nad CR wnl
             ASPIRIN
             MTX-SECOND LINE

             Interval hx: improved

             Order

             Rheumatology consult
             Eye consult
             Physical therapy consult
             EXERCISE
             Med compliance
             Multi-Vits
             Calcium supplements/ diet rich in calcium
             Educate family
             MRI and Dexa--?
             then do RATED SEX...mneumonic counsellin...whatever is applicable



             22-kawasaki disease.....

             < 5yrs of age
             fever, rash, conjunctival injection, cervical lympphadenitis, inflammation of lips and the oral cavity, redness and swelling of the hands
             and feet.
             coronary arteries aneurysm
             unknown etiology.




16 of 43                                                                                                                                          4/16/12 11:27 AM
50 star ccs cases - USMLE Forums                                                        http://www.usmle-forums.com/usmle-step-3-recommended-thre...



             PE ; complete

             Orders:

             pulse ox
             oxygen
             cbc
             bmp
             lfts
             esr
             ua
             12 lead ekg
             bld cxs
             LP
             Urine cx
             CXR
             IVA

             results:
             thrombocytosis
             elevated ESR
             sterile pyuria
             EKG=ST seg depression and T-wave flattening
             mild hypoalbuminemia

             ORDER:
             2-D echo
             Coronary angiogram....maybe???
             Aspirin ( untill pt is afebrile for several days)
             IVIG
             Consult Peads Cardiologist (like Dr. Fisher says on CCS always Consult; it wont harm U)
             should continue 3-5mg/kg/day, d/c after 6-8 weeks if no signs of coronary involvement and practically indefinitely if there is a
             coronary problem.
             Influenza vaccine before starting aspirin to prevent REYE'S Syndrome

             MMR and Varicella to be delayed till 11 months
             INTERVAL HX: PT HAS DEFERVESCED

             I do not know what to do now....maybe...

             ORDER:
             d/c home on aspirin
             f/u in 7 days
             cbc on follow up may be in 30 days to look for platelets
             esr follow up
             repeat ECHO. 6-8 weeks out

             counselling...



             23-Osteomyelitis

             PE; complete..except breast, genitalia and rectal

             * Orders:
             admit

             iv acess
             iv saline
             blood cs
             urine ua and cs

             cbc
             bmp
             pt
             ptt
             ESR
             C-reactive protiens




17 of 43                                                                                                                                        4/16/12 11:27 AM
50 star ccs cases - USMLE Forums                                                          http://www.usmle-forums.com/usmle-step-3-recommended-thre...


             X-ray of the involved leg

             ~Results...x-ray wnl... ESR 90.....

             *Order
             MRI or Bone scan( if MRI is... C/I)..........MRI more specific!
             Bone Biopsy



             ~Results ..... MRI= Mild destruction fo tibia... Bone Bx=GM shows neutrophils & Stph aureu grows on cxs!

             * Admit to ward
             diets
             bedrest with bathrm privilages
             cbc for day #2
             Empiric coverage with Oxacillin & Cipro ...OR.... Ofloxacin & ceftriaxone....treatment for 6 -12 wks....IV...
             Gram negative osteomyelitis treated with Cipro orallay.

             * 5 minute screen

             RATED SEX
             age appropriate tests...

             -----------------
             24-Hypoglycemia
             27 yrs old female nurse found unresponcsive, daiphoretic and tachycardic. Prior to this she wa sc/o headache and tremors. Pupils are
             wnl. PmHx is insignificant.

             diffrential includes

             Insulinoma
             Exogenous Insulin
             SU overdose
             Prolonged fasting

             O2
             pulse ox
             IVA
             Vitals Q 1 HR
             BP
             Cards
             ekg
             CXR
             Accuecheck
             Beta Hcg urine
             thiamine
             dextrose 50
             naltrexone- if pupil constricted

             then iv infusion-5%dex

             ~PE : HEENT, LUNGS, HEART AND ABDOMEN

             *Result BS 50 and pupils are wnl

             ~ORDERS:
             cbc
             bmp
             cal
             mag
             phos
             lfts
             UA
             abg
             C-peptide
             Insulin serum
             Insulin antibodies
             Bld alcohol level
             Urine tox




18 of 43                                                                                                                                      4/16/12 11:27 AM
50 star ccs cases - USMLE Forums                                                       http://www.usmle-forums.com/usmle-step-3-recommended-thre...


             SU urine screen
             TSH
             Cortisol level
             Lipid panel

             ~PE: come back and finish the exam now.

             * Interval histoyr...pt is a little awake

             ~Results: C-peptide rasied and SU urine tscreen +ve for glyburide!

             ~Order: Octreotide SC x 1 bolus

             do ct/ABD to see insulinoma

             gastroenterology surgeon consult for surgey

             * Interval Hx and VItals: improving

             ~Orders:
             Transfere to ICU
             Octreotide sc q 8hrs
             Accue check q 1hrs
             NPO
             Urine output
             teds
             bedrest
             cbc in AM
             bmp in AM
             Psychiatyr consult

             * Interval History/vitals check...

             * improved.. d/c npo, bedrest and octreotide and dextrose.when BS in the range of 85-90
             * Move to the ward....

             ~ "5 MINUTE SCREEN"
             PAP
             Rated SEX ..whatever is applicable
             screening (mamo if age >40)
             colono if age>50
             counsel (I select as many counsellings for all patients as poss)
             diet consult
             suicide contract if OD
             resched visit in another 4 wks
             f/u in 2 weeks after the discharge...



             ================================================== =====
             If insulinoma is suspected..then CT abdomen or USG abdomen..
             DEBULKING i.e surgery is the treatment then.....

             anyone still feels the need to add something...lol...be my guest..
             this is an exhausting one, for sure!




             25-Dilated cardiomyopathy

             55 yr old pt presents w bilateral LE edema, sob on exerction, no cough. h/o drinking 5 quarts of wine every day. PE bibasilar wet
             crackles and evidence of moderate ascites!!!

             Casuses of dialted CM

             alcohol
             adriaamycin
             radiation
             viral myocarditis
             amyloidosis




19 of 43                                                                                                                                         4/16/12 11:27 AM
50 star ccs cases - USMLE Forums                                                     http://www.usmle-forums.com/usmle-step-3-recommended-thre...


             sarcoidosis
             hemachromatosis
             Thiamine deficiency

             ~ Order: put thme in as STAT
             O2
             Pulse OX
             EKG
             CXR
             Cards
             Vitals q 2
             CBC
             BMP
             IVA
             Fingerstick

             PE ; COMPLETE
             Cal
             Mag
             Phos
             Lfts
             Amylase
             Lipase
             UA
             PT
             PTT
             lipid
             tsh
             e cho
             B12
             FOLIC
             BLOOD ALCOH
             HEAD ELEV

             * Results : CXR=Enlarged heart w Kerley B llines + EKG= ST-T waves non-specific abnl

             ~ Admit to floor ~

             ~Order
             IVA
             Low Na diet
             Bedrest w bath rm privilages
             TEDS
             Hepari SC
             Lasix IV
             KCL
             MORPHI
             Foley
             Strict Input and Output
             Daily weights
             Cardiac Echo ...now!
             CXR Q day
             BMP q day

             * interval hx...pt hope meds will help

             ~Result: echo shows dilated heart w EF 25%

             ~Order

             iv carvedilol
             iv spirono
             iv lisino
             iv digi
             Anticoagulation...consider in longterm .... if evidence of thrombosis
             strict daily wt, i/o (foley for strict uo)
             daily mvi, thiamine and folate (commonly deficient)
             bed rest
             low salt diet




20 of 43                                                                                                                        4/16/12 11:27 AM
50 star ccs cases - USMLE Forums                                                       http://www.usmle-forums.com/usmle-step-3-recommended-thre...


             fluid restriction

             statin if abn lipids
             cards consult

             D/C ALCOHOL
             ACE receptor blockers for those who cannot tolerate ACEI

             when stable:
             dc all iv meds-->po—day3
             f/u in 1 wk with another bmp and ekg
             echo in 2-3 wks
             screen
             counsel
             dietician consult

             ~ 5 Minute Screen
             alcohol anonymous
             alcohol abstienance
             lipid
             colonospcopy
             Rated SEX
             RPT ECHO-3MONTHS
             CARD REHAB

             Maybe you will have to manage the pt for 2-3 days in the simulated time...on the software...
             but d/c home on ace, beta bxs, spironolactone, dig and lasix...with follow up in next 7-14 days... Of course change IV meds to PO
             befor discharge



             -------------------------



             26-Down's Syndrome neonate

             new born downs most probably presents with vomitting ( duoenal atresia)

             telemetry

             Pulse oxy
             IVA
             iv one fourth normal saline
             oxygen
             BP monitoring
             Cardiac MONITOR
             NPO
             NGT
             IV metoclopramide stat

             Brief physical

             ABG
             CBC
             BMP
             CXR
             EKG
             AXR acute series
             USG abd
             UA
             urin culture
             LFTs
             amylase
             lipase

             USG confirms the Diagnosis
             Consult Pediatrics GI surgery

             Transfer to ICU




21 of 43                                                                                                                                         4/16/12 11:27 AM
50 star ccs cases - USMLE Forums                                                     http://www.usmle-forums.com/usmle-step-3-recommended-thre...


             vitals Q 1 hr
             NPO
             Urin out put
             karyotyping
             BP check
             electrolytes

             Karyotype confirms Down's

             ECHO
             audiometry
             TSH

             Genetics consult
             -------------------------------------

             27-Preeclampsia

             21 yr 0ld at 33wks gestation, c/o facial and upperextremity edema....
             how will you proceed

             pulse oxy
             IVA
             BP monitor
             Cardiac Monitor

             brief physical



             CBC
             BMP
             LFTs
             blood typr n cross match ( if not done already)
             pt/ptt
             UA
             Obtretical USG
             DEXAMETH
             MGSO4-IM[deli/labo]

             IF SEVERE PRE-ecl- Im-MgSO4 stat-cont
             IV hydralazine stat-cont

             ECL IV MgSO4 stat-cont
             IV hydralazine stat-con

             Urin creatinine ( it will be included in UA I guess)

             Transfer to ward

             interval hx-check-neuro
             pulse oxy Q 2hrs
             Vitals Q 2 hrs
             urin output
             bp check
             complete bed rest
             serum Mg Q 4 hrs-if give
             Urin 24 hr uric acid
             FHR monitoring
             fetal doppler
             OB consult

             observe for 24 hrs
             pt gets better



             deliver the baby after –term-nvd with oxy



             ------------------------




22 of 43                                                                                                                        4/16/12 11:27 AM
50 star ccs cases - USMLE Forums                                                         http://www.usmle-forums.com/usmle-step-3-recommended-thre...


             28-MM

             Multiple Myeloma** patient presenting with Hypercalcemia

             CCS: hypecalcemia in a pt who is presenting acutely...... lets say a 45 year old male presents with abdominal pain, wife reports he has
             been acting a bit strange over the last few days.... how would you proceed.... Presented in emergency.!

             Differential will be following:
             90% Primary Hyperparathyroidism
             Malignancy---
             1-* Osteolytic Hypercalcemia due to Myeloma, Lymphoma, Breast carcinoma
             2-*Humoral Hypercalcemia-PtHrP – Sq Cell Cacinoma of lungs, head & neck, renal or bladder.
             3-*Tumoral Calcitriol production is Hodgkins & Non Hodgkin Lymphomas.

             ~ORDERS:
             Iv Access
             Pulse oximetry
             Oxygen inhalation continuous
             Vitals x 1hr

             Focused PE : General, HEENT , Heart , Lung, Abdomen, Extremities
             ~ORDERS:
             cbc- ----- Stat
             bmp ------Stat
             calcium--- Stat
             Mag ---Stat
             Phos ---Stat
             Lfts ---Stat
             UA ---Stat
             EKG ---Stat
             CXR ---Stat

             Move the clock get the result
             ~*Results show Ca 13.5mg/dl
             ~*Results show Hb 8.7g/dl . Also BUN and Creatinine slightly Increased
             ~* EKG shows shortened QT Interval

             ~ORDER;

             Iv 0.9% Saline Continuous
             Salmon Calcitonin - SC q 6-12 hours
             Or IV Pamidronate Continuos (over 2-6 hours0
             PTH assay
             24 hr urinary calcium
             Sulfosalicylic Acid Test (to detect ~*Bence Jones Protein… coz Normal urine dipsticks will not detect light chain)
             Seum Protein Electrophoresis
             Urine Protein electrophoresis
             Seum alkaline phosphatase (to rule our Hyperparathyroidism & Paget disease)
             Serum Ferritin
             TIBC
             Serum Iron
             Ultrasound of Abdomen (to rule out Renal carcinoma)

             ~*Do Interval and Check for volume overload by focused Heart & lungs

             ~ORDER;

             If Volume overload then give
             Iv Furosemide One time only

             *******Patients feels better so move the patient to Ward
             ~ORDER;

             Vitals q 2 hours
             Iron Enriched Diet
             Ambulate at will
             Urine output
             BMP daily
             Sodium Docusate (stool softner)




23 of 43                                                                                                                                       4/16/12 11:27 AM
50 star ccs cases - USMLE Forums                                                        http://www.usmle-forums.com/usmle-step-3-recommended-thre...


             Calcium Daily
             Bone Xrays

             Move the clock … get results
             ~* Results show Bone X ray *Lytic Lesions.
             ~* Results show SPEP and UPEP *Positive
             ~* Results show PTH .*Normal”

             Order *Bone Marrow Biopsy
             Consent for Procedure
             Move the clock …Get the biopsy result ~* which shows “Plasma cells” in Bone Marrow

             ~*Call Oncology consult... Patient with Multiple Myeloma needs chemotherapy
             ~ORDER;
             Chemotherapy ---Vincristine, Adriamycin, Dexamaethasone.

             Hopefully case ends here
             5 minute screen
             do all the counselling
             and age appropriate test.

             Epogen** SC to fix his anemia----- erythropoietin



             -------------------------




             29-Hypercalcemia-off ice

             30 yr old with hyper calcemia on routine tests with h/o renal stones some 2-3 yrs bcak. Serum calcium is 11.5 mg/dl.

             PE complete except breast, recal and genitalia

             ~ORDERS:
             cbc
             bmp
             calcium
             Mag
             Phos
             Lfts
             PT
             PTT
             UA
             EKG
             CXR

             send the pt home..call him when all the lab result return!

             Pt returns...do a small PE
             Results show ca 11.5mmg/dl

             ORDER;
             PTH assay
             24 hr urinary calcium
             Dexa Scan

             pt home, call with results of the tests

             Results show..PTH 23 mEq/ml

             Call general surgery consult...parathyroidectomy.....
             as pt fulfills 2 criteria for undergo parathyroidectomy + age less than 50 and h/o Renal stones..

             Hopefully case ends here

             5 minute screen
             do all the counselling




24 of 43                                                                                                                            4/16/12 11:27 AM
50 star ccs cases - USMLE Forums                                                        http://www.usmle-forums.com/usmle-step-3-recommended-thre...


             and age appropriate test.

             -------------------------
             30-Hypercalcemia-ER

             a 45 year old male presents with abdominal pain, wife reports he has been ating a bit strange over the last few days....

             iva
             pulse oxi stat
             vitals q1h
             cardiac monitor
             cxr
             abdo usg
             cbc stat
             bmp stat
             ca, mg, phos stat
             ua stat
             amylase stat
             lipase
             lfts
             tsh
             pt/ptt
             morphine iv

             full physical

             npo
             sr. pth
             24 hr ur. ca
             spep/upep
             vit d level
             abdo ct
             normal saline iv
             furosemide iv
             calcitonin

             if better:
             tt ward

             vitals q4h
             bmp, ca, mg, phos q4h
             consult surg if pth for parathoidectomy (blood type and cross match, npo, foley)

             dc all iv's
             alendronate

             send home after 48 hr
             f/u in 1 wk with cbc, bmp, ca, mg, phos, sr. pth

             5 min screen:
             colono
             counsel
             f/u in 4 wks with same labs

             ----------------------




             31-MS

             24 yr F pt comes to your office with blurring of vision in L eye and weakness of right leg. Past history of such weakness episode on R
             arm prior to 6 months.--ER

             PE(complete)

             fundoscopy shows blurring of disc margin, disc atrophy;

             admit to ward




25 of 43                                                                                                                                        4/16/12 11:27 AM
50 star ccs cases - USMLE Forums                                                  http://www.usmle-forums.com/usmle-step-3-recommended-thre...


             Pregnancy test
             cbc
             bmp
             ua
             tsh
             vit b12
             pt/ptt

             mri brain and spine( shows demylinating lesions)
             iv methyl prednisolone

             consult neurology
             consult opthal
             LP( shows oligoclonal bands)

             neuro check-2h

             baclofen for spasticity
             if pain gabapentin
             bladder hyperactivity oxybutinin
             fatigue amantadine or flouxetine
             urinary retention bethanecol
             Contracpetive

             normal diet/urine output/ambulation at will
             r/w after 12 hrs
             interval history and PE

             symptom free-- on day3

             d/c IV methyl pred
             Fasting blood sugar
             cbc
             bmp
             calcium
             vitaminD

             Interferon or Glatimer acetate start
             counsel patient
             contraception
             oral predni
             vacc-influ
             eye consult

             follow up appointment; after 3 months MRI repeat

             ------------------------

             32-HELLP

             35 wks getation...bp 170/115, headache, scotoma, epigastic pain-ER



             pulse ox
             Oxygen
             cardiac monitors
             Bp continous
             IV access
             NS
             NPO

             MgSO4..IV
             Latetalol..IV
             DEXAmethasone.. IM

             P/E

             Admit to ICU...




26 of 43                                                                                                                     4/16/12 11:27 AM
50 star ccs cases - USMLE Forums                                                       http://www.usmle-forums.com/usmle-step-3-recommended-thre...


             BED
             U OUT
             PNEU
             RANITIDINE-IV

             CBC
             BMP
             Ca, Mg. Phos
             LFTs----------------CONSULT OB
             PT/PTT
             Retic
             FSPs
             D-Dimers
             UA
             USS-PELV
             Labor monitor
             FETAL MONITOR
             CULTURE-VAG,GONO,CHLAM

             MONITOR—BP,PLT,UA-PROTEIN,PT.PTT,BMP

             type and cross 2 units of PRBCs
             FFP
             Platelet

             PENICILLIN G-IV
             OXYTOCIN
             NVD

             5MIN
             H&H
             ORAL DIET
             PT
             PTT
             OMEPRA
             BF
             PAP
             LFT



             ----------------------------------
             33-Endometrial Hyperplasia without Atypia

             hx if imp for age, if she wants childbirth or contraception.
             usually presents with abnormal heavy uterine bleed. (if very heavy: send to ER), lets assume here its moderate, no distress, pt now in
             office.

             full PE

             CBC
             BMP
             UA
             LFTS
             B-HCG
             TSH
             PT/PTT
             BLEEDING TIME
             PAP
             ENDO BX
             USG-PELVIS-ENDO THICKNESS

             Call her again in 2 days:
             if anemic, order FOBT, RETIC, PERI SMEAR, TIBC, SR. IRON, FERR

             RESULTS ALL WNL. BX SHOWS HYPERPLASIA WITHOUT ATYPIA



             CYCLIC-
             MONOPHASIC OCP'S IF SHE DESIRES CONTRACEPTION




27 of 43                                                                                                                                      4/16/12 11:27 AM
50 star ccs cases - USMLE Forums                                                      http://www.usmle-forums.com/usmle-step-3-recommended-thre...


             OR
             MEDROXYPROGESTERONE ORAL FOR 14 DAYS IF NOT
             FESO4
             MVI

             Call back in 14 days
             IMPROVED BLEEDING: NO SYMPTOMS: CAN DC MEDROXY
             IF PERSISTENT: CONTINUE MEDROXY FOR 6 MONTHS

             Call in 1 month to check

             5 min screen:
             pap x1 yr
             mammo
             endo bx x6months
             Colono
             counsel

             IF OLD PATIENT: SEVERE BLEEDING: HYSTERECTOMY



             --------




             34-Gastric cancer

             Weight loss , Abdominal pain , Nausea, presntation was Gastric obstruction
             Dysphagia, Melena, Early satiety ,Ulcer-type pain

             NPO
             NG Tube
             IV Fluid
             Vitals
             Iva
             Pulse oxy
             ABG
             CARDIAC MONITOR

             CBC
             BMP
             UA
             CXR
             EKG
             XRAY-ABD-series

             P/E

             FOBT
             LFT
             LIP ID
             FBS
             PT
             PTT
             ALBUMIN
             CA,MG,PHOS
             AMYLASE
             LIPASE
             PHENERGAN
             MORPHINE

             WD

             OUTPUT
             Barium upper GI studies
             CONSENT
             CONSULT-GASTRO
             ENDOSCOPY
             BIOPSY-NO OPTIONIN SOFTWARE




28 of 43                                                                                                                         4/16/12 11:27 AM
50 star ccs cases - USMLE Forums                                                           http://www.usmle-forums.com/usmle-step-3-recommended-thre...


             COLONOSCOPY

             ENDOSCOPY,BX
             ADENO CA

             ANEMIA-IRON STUDIES,VITB12
             FESO4, VITA C

             CT-CHEST
             CT-ABD
             PT
             PTT
             TYPE

             NEXT ORDER

             CONSENT
             CONSULT –GASTRO-SX
             CONSULT-RADIO
             CONSULT-ONCO
             CONSULT-DIETICIAN

             ADVANCE DIRECT
             --------------------------------------------



             35-TURP-HYPONATRE MIA

             Manifestations of the TURP Syndrome:
             • - acute hypo-osmolality
             • - acute hyponatremia
             • - congestive heart failure
             • - pulmonary edema
             • - hypertension
             • - hypotension
             • - solutee toxicity:
             hyperglycinaemia (glycine)
             hyperammonaemia (glycine

             if detected intra-operatively bleeding points should be coagulated, surgery
             terminated as soon as possible and iv.fluid should be stopped



             OXY
             VITALS
             CARDIO
             BP MONIT
             OXY

             FOLEY
             CULTURE-BLOOD/URINE
             CBC
             BMP
             UA
             EKG
             CXR

             DC-NSS
             DC-SX
             FUROSEMIDE-IV-AFTER LOW Na



             ICU

             BED
             NPO
             OUT
             PT
             PTT




29 of 43                                                                                                                              4/16/12 11:27 AM
50 star ccs cases - USMLE Forums                                                         http://www.usmle-forums.com/usmle-step-3-recommended-thre...


             CBC
             HandH
             TYPE
             ABG
             PULSE OXY

             IF SEIZURE/confusion-
             DIAZEPAM
             3% NACL

             -----------------------------



             36-testicular pain-acute unilateral, child--ER

             testicular torsion, - the most dramatic and potentially serious of the acute processes
             torsion of the appendix testis,-MC
             epididymitis.



             abrupt onset of severe testicular or scrotal pain.N,V
             awaken with scrotal pain in the middle of the night or in the morning

             VITALS-FEVER NO
             IVA
             OXY
             CARDIO
             BP MONITOR

             CBC
             BMP
             UA/culture

             PHENERGAN
             MORPHIN

             P/E-
             GENTAL,ABD�� �HORIZONT LEVEL
             CREMESTERIC REFLEX-NEG

             NPO
             TRANSILLUMINATION
             USS-SCROTUM--------TEST:TORSION

             CX R
             amylase
             lipase
             LFTs
             PT
             PTT
             TYPE
             CONSULT-PED SX
             Cefazolin
             B/L ORCHEOPEXY
             Orchiectomy is performed if the testicle is nonviable




             IF FEVER+POSITIVE CREMESTER WITH NEG USS---ACUTE EPIDIDYMITIS
             frequency, dysuria, urethral discharge –UTI
             CULTURE

             CHILD WTH UTI-CEPHALEXIN,BACTRIM

             GONO- CEFTR+DOXY

             scrotal support, rest
             ----------------------------------------------




30 of 43                                                                                                                            4/16/12 11:27 AM
50 star ccs cases - USMLE Forums                                                        http://www.usmle-forums.com/usmle-step-3-recommended-thre...




             37-hype rtensive crisis with SAH

             Ischemic stroke or subarachnoid or intracerebral hemorrhage -- Intravenous labetalol Other first-line agents include transdermal
             nitroglycerin paste and intravenous nicardipine
             nitroprusside should be considered second-line therapy
             Sublingual nifedipine should be avoided

             o Antihypertensive agents previously were advocated for an SBP greater than 160 mm Hg or diastolic BP (DBP) greater than 90 mm
             Hg.
             o Keep systolic blood pressure 90-140 mm Hg before aneurysm treatment, then allow hypertension to keep systolic blood pressure
             less than 200 mm Hg




             Acute pulmonary edema -- Nitroprusside or nitroglycerin with a loop diuretic
             Drugs that increase cardiac work (hydralazine) or decrease cardiac contractility (labetalol or other beta blocker) should be avoided
             Angina pectoris or acute myocardial infarction - nitroprusside and nitroglycerin
             Aortic dissection - beta blocker such as propranolol or labetalol. +/- Nitroprusside

             Noncontrast brain CT or brain MRI
             Electrocardiogram
             Complete blood count including platelets
             Cardiac enzymes and troponin
             Electrolytes, urea nitrogen, creatinine
             Serum glucose
             Prothrombin time and international normalized ratio (INR)
             Partial thromboplastin time
             Oxygen saturation
             Lipid profile

             Lumbar puncture if subarachnoid hemorrhage is suspected and head CT scan is negative for blood Electroencephalogram if seizures are
             suspected
             ------------------------------------

             38-Hypertension-secondary

             youn g man no risk factor

             cbc
             bmp
             lft
             pt
             ptt inr
             lipid
             tsh
             ua
             uds
             cxr
             ekg
             FBS

             home

             if sodium high potassium lo normal anion gap

             give KCL

             office
             go for aldesterone/rennin activity ratio
             if high-24 aldosterone level
             spiranolactone
             abd/ct




31 of 43                                                                                                                                        4/16/12 11:27 AM
50 star ccs cases - USMLE Forums                http://www.usmle-forums.com/usmle-step-3-recommended-thre...


             monitor BMp

             home
             ct-adrenal mass

             ward

             consent
             consult
             pt
             ptt
             type
             npo
             laparoscopic adrenalectomy
             --------------------------------
             if cxr show cardimeg

             rib notching-MRA

             go for coarctation

             if ua proteinuria

             do ultrasound kidney
             mara kidney

             do nephro consult
             do surgey consult



             measure bp in both arm

             start meds

             beata-2
             hctz-1
             acei
             ccb
             smoking
             obesity
             alcohol
             drug hx

             coumsel
             call back

             --------------------------



             39-fever unknown origin - adult



             h pe

             cbc
             bmp
             lft

             pt ptt inr

             cxr

             ekg

             ua

             uds if prtinent

             blood culture




32 of 43                                                                                   4/16/12 11:27 AM
50 star ccs cases - USMLE Forums                            http://www.usmle-forums.com/usmle-step-3-recommended-thre...



             urine cs
             sputum

             lp if meningitis suspected or alter mental



             ct if necessary

             gyn consult -- if gyn cause

             surgery if abscess

             if lft increse

             hepatitis panel



             if central lines line culture

             if janeway osler roth spot

             or bacterial endo--echo

             if throat pain lad

             mono

             atypical lypho in blood

             rapid strep test

             if viral syndrome

             vdrl hiv

             if leg pain or sob or pe suspected
             vq scan

             later -doppler

             or low -d dimer

             if no improve -



             joint lymph node ry eye dry mouth

             connective tissue panel

             treatwith

             abx

             iv saline

             npo if sepsis suspected

             surgical consult and ct if abscess

             see the bllod cs report change the abx

             no response you may have to add amphotericin

             if herpes thing - add acyclovir

             -----------------------------------------

             40-septic shock




33 of 43                                                                                               4/16/12 11:27 AM
50 star ccs cases - USMLE Forums                                                          http://www.usmle-forums.com/usmle-step-3-recommended-thre...



             fever
             shock

             tachy
             low bp
             left lowe quadrant pain

             iv normal saline
             iv access
             oxygen
             pulse oxy
             later abg

             focus pe

             cbc
             lft
             pt ptt
             ua uds
             urine culture
             blood culture times two

             ct abdomen and pelvis with contreast
             lactic acid
             cxr
             echo if bacterial endocarditis suspected--later in floor if he does not improve
             can do cardiac enzyme to rule out cardio shock
             amylase lipase
             xray abdomen - do or dont do because yo do ct anyway

             start abx
             cefotaxime
             genta
             intake out put
             foley
             urine out put 1 hourly

             pt get bette -continue

             ct reort comes
             surgical consult for drainage of abscess

             better

             dc with cipro
             -------------------------------
             41-Alcohol withdrawal

             tachy,sweating,tremor,agitated

             iva
             o2
             pULSE OX
             cARDS MONITORS
             bP
             Accue check

             PE... real quick
             HEENT
             RESP
             CARDS

             Labs... STAT..
             npo
             nss
             cbc
             bmp
             lfts




34 of 43                                                                                                                             4/16/12 11:27 AM
50 star ccs cases - USMLE Forums                                     http://www.usmle-forums.com/usmle-step-3-recommended-thre...


             Blood alcohol level
             Urine toxicology
             ABG
             amylase
             lipase
             PT
             PTT
             Thiamine IV
             Folic acid Iv
             Calcium serum
             Mag srum
             Phos serum



             now come back and Complete the PE...what ever is lfet

             librium
             transfere to ICU

             seizure precaution
             aspiration precaution



             5 minute screen
             counselling
             RATED SEX...
             alcohol anonymous..
             -----------------------------------

             42-retained placenta

             (ER,3post op D, w fever and abd tenderness)



             Pulse Oxymetry
             IV Access
             IV NSS

             Complete PE

             CBC with Diff
             PT/PTT
             Blood Grouping and Cross Match
             LFTs
             UA and CS
             Blood Cultures
             Cervical Cultures and gram staining

             IV Ceftriaxzone
             IV Clindamycin

             Consult OBG, for retained placental removal

             send the pt to medical ward:

             Bed rest
             NPO

             Vitals Q4H
             ----------------------------------

             43-Chronic renal failure



             in office



             take complete physical exam




35 of 43                                                                                                        4/16/12 11:27 AM
50 star ccs cases - USMLE Forums                                                        http://www.usmle-forums.com/usmle-step-3-recommended-thre...


             order
             vitals
             cbc
             bmp
             ua
             cxt
             abd ultra sound
             abg
             ekg
             serum lipids
             serum albumin
             serum calcium
             phosphate
             vitamin D
             pth
             lft

             if pt in emergency

             then we do iva and also see the urnie output

             check phosphate lever

             and also pt ptt
             and do blood typing
             as tehre might be anaemia

             so we mite need transfusion

             or even dialysis if acidosis

             calcium

             tratment is diet

             ----------------------------------

             44-Acute renal failure

             80 yr old man comes to ER with n/v and maliase. PMhx is significant for Htn, DM and osteoarthritis. Pt is on NSAIDS, lisinopril. Also
             reports making very little urine over the last 24 hrs.




             PE : General, heent, LUNGS, ABDOMEN

             ~ Orders:

             iva
             nss
             pulse ox
             vitals Q 2 hrs
             cards
             ABG
             Accue check
             ekg and cxr
             CBC
             BMP
             Cal
             Mag
             Phos
             UA
             Urine cxs
             LFTs
             tylenol

             ~PE: come back and complete the rest of the exam now...

             *!* Results..(.pH 7.29, PCO2 20, PO2 80). (BMP NA 138 & HCO3 12)




36 of 43                                                                                                                                        4/16/12 11:27 AM
50 star ccs cases - USMLE Forums                                                      http://www.usmle-forums.com/usmle-step-3-recommended-thre...



             ~Transfere to ward

             ~Order

             low potassium diet/diabetic diet
             bed rest with bathroom privilages

             d/c NSAIDS
             d/c Lisinopril

             FOLEY
             Strict input/putput
             Teds

             Urine NA ( NL IS LESS THAN & EQUAL TO 10) and Creatnine
             24 hr Urine protien
             eosinnophils in urine...(seen in allergic nephritis)
             Renal USG ( if BPH...call urology consult)

             Hgb A1c
             DAILY WEIGHTS
             Accue check q 4 hrs
             Insulin sliding scale ( if need be )
             BMP q 2-4 hrs

             hopefully pt starts to improve after d/c nsaids and NSS infusion...
             pre-renal RF treated with fluids...if no rsponse...IV lasix....
             Dobutamine and dopamine (if heart failure)

             ~famous 5 minute screen
             RATED SEX...what ever is applicable.

             ---------------------------------------------

             45-rape

             complete physical

             orders:
             maybe one ste of vitals..
             RAPE KIT...
             cbc..for baseline
             UA
             pregnancy test..beta HCG urine
             cervical smear
             KOH prep
             Hanging drop
             cervical gram stain and culturegonorrhoe DNA probe testing
             Chlamydia DNA probe testing

             morning after pill..i think it is LEVONORGESTREL-oral(high dose estrogen) for 2days
             Now STD prophylaxis:
             Ceftriiaxone 125 mg IM
             Azithromax 1gm PO
             Probenecid
             Metonidazole 2gm po for trichomonas

             add vdrl
             rape crisis consult
             cervical sample for chlamy and gonorr
             elisa for HIV
             Hep B surface antigen

             Social services consult
             Psych Consult?
             ----------------

             46- HUS




37 of 43                                                                                                                         4/16/12 11:27 AM
50 star ccs cases - USMLE Forums                                                    http://www.usmle-forums.com/usmle-step-3-recommended-thre...



             ER
             Focused PE

             IVA
             NSS
             Oxygen
             pulse oxymetry
             Monitor Blood Pressure

             NPO
             CBC
             BMP
             UA, Culture
             AXR acute series
             Stool leuco, cultu, ova & para
             peripheral smear
             ldh
             haptoglonin
             in/output
             pt/inr
             pt/ptt
             blood type cross match
             D-DIMER
             FIBRIN DEGRADATION PRODUCTS-fdp

             results come as low platelet
             fragmented RBC
             no FDP
             no D dimers
             PT/PTT are normal
             K+ is elevated

             treatment with keyexalate

             Once stable transfer to ICU
             monitor BP
             cbc
             check BMP again every 1 hr till K+ normalizes followed by every 4hrs
             pt/ptt
             supporitve for now.
             consult hema pediac
             counsel pat/fam

             If case doesn't improve plasmapheresis
             ***check for ldh inc. schisto in peripheral, retic increa
             BUN & crea are in BMP
             ----------------

             47-New onset DM-42 yr old c polydipsia & polyuria

             DD- DM, DI, Factitious Disorder

             since it is a clinic setting...no emergency..

             Pe: complete

             ORDERS:
             cbc
             bmp
             ca, mg, phos
             UA
             12ekg
             ABG
             lipid profile
             cxr
             HgbAIC
             lft




38 of 43                                                                                                                       4/16/12 11:27 AM
50 star ccs cases - USMLE Forums                                                       http://www.usmle-forums.com/usmle-step-3-recommended-thre...


             Finger stick BS 325mg/dL.—DIAGNOSTIC DM

             Admit to inpx service

             monofilament skin test,
             serum/urine ketones,
             serum/urine osmolarity,
             urine microalbumin.
             UA-CULTURE
             -ivf nss,

             -sliding scale:bld glucose 100-200-do nothing
             bld glucose 200-300-5u insulin
             bld glucose 300-400-10u insulin

             blood glucose (accucheck or finger stick) q2h,
             BMP-2h
             -vitals q4h,
             Activity prn,
             foley's catheter for intake
             output,1800 ADA (55-60% cho,less tham 30% fat,15-20% protein,vitamins,minerals,H20),

             lisinopril and other anti HTN (if HTN)
             -podiatry or chiropody consult
             -endocrinilogy consult
             -ophthalmology consult

             -If insulin 100-200,dc insulin and institute glipizide,
             dc ivf
             -counsel:weight loss,diet,exercise,
             annual ophthalmologic exam,
             foot care and protection,medication compliance,
             depression couseling,
             family counseling,medication side effect counseling,annual health maintenance and flu vaccine couseling.
             -follow-ups.



             --------------------



             48-Fever Unknown origin-child

             INFANT-bac,HSV
             Child-infect,connective

             Cbc
             Ua
             Bmp
             Cu lture-blood,urine,throat
             Lft
             Ana
             Rf
             Esr
             Cxr
             Lp-irritable

             Ppd
             Hiv
             Syph

             Ct-abd

             Wbc scan-gallium/indium

             NO-emp..ABx
             --------------------------------------- -----------

             49-Cervical cancer




39 of 43                                                                                                                          4/16/12 11:27 AM
50 star ccs cases - USMLE Forums                                                      http://www.usmle-forums.com/usmle-step-3-recommended-thre...


             physical exam

             cbc
             bmp
             ua
             urine beta hcg
             gonorhea probe
             chlamydia probe
             wet mount
             vaginal ph
             HIV ELISA
             VDRL
             pap

             move the clock forward
             if has come with chlamydial infection/ginorrhea treat that

             call in 3 days (pap result comes in 3 days)

             Colposcopy
             Endocervical curretage(its there in the software but asks for Gynecolgy consult)
             Gynecology consult(No Endocervical biopsy on the software so can ask for that also on the Gynec consult)

             Move the clock forward

             Call her in a week

             colpo-cx ca

             Interval history
             Admit to ward

             Bed rest with bathroom previleges
             cxr
             lft
             pelvic ex
             IVP
             CSYTOSCOPY
             SIGMOIDOSCOPY
             abdominal ct
             pelvic ct
             bone scan

             RADIO-CONS
             oncology consult
             ekg 12 leads
             blood type cross match
             pt
             ptt
             Serum Iron with TIBC
             reticulocyte count




             interval history

             TAH+BSO(If family done)
             Gynecology consult
             RADIATION/CHEMO-CISPLATIN

             patient education
             no smoking
             no alcohol
             supportive psychotherapy
             Iron enriched diet

             50-Failure to Thrive

             2yr - below 5th percent




40 of 43                                                                                                                         4/16/12 11:27 AM
50 star ccs cases - USMLE Forums                                                                   http://www.usmle-forums.com/usmle-step-3-recommended-thre...




             If severe malnutrition/abuse---Hospitalization

             dd-
             low intake
             abuse
             chd
             infection
             endo
             genetical

             Hx+EX

             Head,neck,weight
             Cbc
             Bmp
             Ua
             Cxr
             Fobt
             Lft
             Culture-stool/urine
             Folic
             Vit b12
             Stool-ova/fat/

             --
             HIV
             PPD
             TSH
             Sweat test
             Galactose-----

             ---
             Caloric count
             Nutrient supp
             Cons-dietician
             Social service



             F/u-q week




            The Following 10 Users Say Thank You to navz For This Useful Post:
             angelina vivien (04-04-2011), chatti (09-02-2011), dr.dhruvdesai (06-16-2011), drrsahuja (11-12-2011), jahn77 (2 Weeks Ago),
             jatnpatl (05-18-2010), missmbbs (11-27-2010), mle_asap (05-05-2010), shaan (07-14-2010), tommylee (02-24-2012)


                              Foods To Lower Cholesterol Healthy diets and tips. Find foods to lower cholesterol. BestHealthDiets.com
                              Ask a Urologist Online A Urologist Will Assist You Now! Questions Answered Every 9 Seconds. Health.JustAnswer.com/Urology
                              Test Your PT/INR at Home Check your own warfarin levels with Philips PT/INR Self Testing at home inrselftest.com


              09-23-2009                                                                                                                                          #2

                                                                                                                                            Steps History: Step 1 Only
                                                                                                                                            Posts: 5
             carroline                                                                                                                      Threads: 0
             USMLE Forums Newbie                                                                                                            Thanked 1 Time in 1 Post
                                                                                                                                            Reputation: 11




             woow
             I kept scrolling and scrolling down and this post never finish
             how reliable is this info!




41 of 43                                                                                                                                                       4/16/12 11:27 AM
50 star ccs cases - USMLE Forums                                                           http://www.usmle-forums.com/usmle-step-3-recommended-thre...


              09-23-2009                                                                                                                                 #3

                                                                                                                                   Steps History: 1+CK+CS
                                                                                                                                   Posts: 3
             navz                                                                                                                  Threads: 1
             USMLE Forums Newbie                                                                                                   Thanked 10 Times in 1 Post
                                                                                                                                   Reputation: 20

                :)



             hahaha i think it is pretty short for 50 cases..bt i think it covers some vital stuff u that cud help u for sure while working wid uw stuff!!!




              11-27-2010                                                                                                                                 #4

                                                                                                                                   Steps History: 1+CK+CS
                                                                                                                                   Posts: 31
                           missmbbs                                                                                                Threads: 1
                           USMLE Forums Scout                                                                                      Thanked 9 Times in 8 Posts
                                                                                                                                   Reputation: 19


                Thanks!


             thanks so much for the post! much appreciated.




              09-02-2011                                                                                                                                 #5

                                                                                                                                   Steps History: 1+CK+CS
                                                                                                                                   Posts: 1
             chatti                                                                                                                Threads: 0
             USMLE Forums Newbie                                                                                                   Thanked 0 Times in 0 Posts
                                                                                                                                   Reputation: 10

                100 important USMLE STEP 3 CCS cases


             that's helpful

             this is a list of dr.red 100 important CCS cases , i think it will help
             http://www.slideshare.net/usmlegalaxy , in this see the dr.red pdf CCS cases




              11-11-2011                                                                                                                                 #6

                                                                                                                                   Steps History: 1+CK+CS+3
                                                                                                                                   Posts: 64
             slime66                                                                                                               Threads: 1
             USMLE Forums Scout                                                                                                    Thanked 3 Times in 3 Posts
                                                                                                                                   Reputation: 13

                Thanks


             WOW!!! I am speechless. And Thanks.




            Tags

             Step-3-Preparation




             Similar Threads

                                      Thread                             Thread Starter               Forum             Replies           Last Post




42 of 43                                                                                                                                               4/16/12 11:27 AM
50 star ccs cases - USMLE Forums                                                                             http://www.usmle-forums.com/usmle-step-3-recommended-thre...


             Similar Threads

             Additional new cases in Third edition of FA                           frankly                    USMLE Step 2 CS Forum             8           11-14-2011 07:42 AM

             Step 2 CS complete and incomplete cases                               ath.pantelis               USMLE Step 2 CS Forum             1           05-30-2010 07:34 AM

             Should we do UW Practice Cases!                                       MikheilGM                  USMLE Step 2 CS Forum             4           01-17-2010 07:03 PM

             Pediatric cases!                                                      chikawawa                  USMLE Step 2 CS Forum             2           07-10-2009 09:10 PM




                                Watch Full Episodes Turn Your Computer into a TV! Watch Full TV Episodes Online. www.TelevisionFanatic.com
                                USMLE STEP 2 CS Review 5 Day LIVE Workshop To Prepare For Step 2 CS Exam... 99% PASS Rate! www.ximedus.com
                                File for SSI Disability Find Out If You Qualify For Social Security Benefits. Free Evaluation! socialsecuritylawfirms.com/denied


                                                           Home - Contact Us - Forum Rules - Privacy - Archive - Top




                                                           USMLE® & other trade marks belong to their respective owners, read full disclaimer
                                                              USMLE Forums created under Creative Commons 3.0 License. (2009-2012)




43 of 43                                                                                                                                                                 4/16/12 11:27 AM

More Related Content

What's hot

Cardiac arrythmia in children
Cardiac arrythmia in childrenCardiac arrythmia in children
Cardiac arrythmia in childrenDr Anand Singh
 
Neonatal cardiac emergency
Neonatal cardiac emergencyNeonatal cardiac emergency
Neonatal cardiac emergencyNeeraj Aggarwal
 
Clinical approach to congenital heart disease diagnosis
Clinical approach to congenital heart disease diagnosisClinical approach to congenital heart disease diagnosis
Clinical approach to congenital heart disease diagnosisikramdr01
 
Systematic ECG Interpretation
Systematic ECG InterpretationSystematic ECG Interpretation
Systematic ECG InterpretationSCGH ED CME
 
pediatrics.Cardiac arrhythmias.(dr.hader)
pediatrics.Cardiac arrhythmias.(dr.hader)pediatrics.Cardiac arrhythmias.(dr.hader)
pediatrics.Cardiac arrhythmias.(dr.hader)student
 
Significance of avR lead
Significance of avR leadSignificance of avR lead
Significance of avR leadJaya Pradhap
 
Approach to child with congenital heart disease
Approach to child with congenital heart diseaseApproach to child with congenital heart disease
Approach to child with congenital heart diseaseAnkur Puri
 
Approach to a Child with Congenital Heart Disese
Approach to a Child with Congenital Heart DiseseApproach to a Child with Congenital Heart Disese
Approach to a Child with Congenital Heart DiseseCSN Vittal
 
The ECG as a Diagnostic Tool: A Clinical Case Study of Supraventricular Tachy...
The ECG as a Diagnostic Tool: A Clinical Case Study of Supraventricular Tachy...The ECG as a Diagnostic Tool: A Clinical Case Study of Supraventricular Tachy...
The ECG as a Diagnostic Tool: A Clinical Case Study of Supraventricular Tachy...Jamie Ranse
 
Pediatric case presentation (congenital heart disease- PDA)
Pediatric case presentation (congenital heart disease- PDA)Pediatric case presentation (congenital heart disease- PDA)
Pediatric case presentation (congenital heart disease- PDA)sakib_lostvalley
 

What's hot (20)

Tachyarrhythmias
TachyarrhythmiasTachyarrhythmias
Tachyarrhythmias
 
Ecg quiz @ SEMICON 1018
Ecg quiz @ SEMICON 1018Ecg quiz @ SEMICON 1018
Ecg quiz @ SEMICON 1018
 
Archer USMLE Step 3 Nephrology 2019
Archer USMLE Step 3 Nephrology 2019Archer USMLE Step 3 Nephrology 2019
Archer USMLE Step 3 Nephrology 2019
 
Cardiac arrythmia in children
Cardiac arrythmia in childrenCardiac arrythmia in children
Cardiac arrythmia in children
 
Archer Neurology for USMLE Step 3
Archer Neurology for USMLE Step 3Archer Neurology for USMLE Step 3
Archer Neurology for USMLE Step 3
 
Neonatal cardiac emergency
Neonatal cardiac emergencyNeonatal cardiac emergency
Neonatal cardiac emergency
 
Clinical approach to congenital heart disease diagnosis
Clinical approach to congenital heart disease diagnosisClinical approach to congenital heart disease diagnosis
Clinical approach to congenital heart disease diagnosis
 
Pulmonology critical care- archer step3 lectures
Pulmonology  critical care- archer step3 lecturesPulmonology  critical care- archer step3 lectures
Pulmonology critical care- archer step3 lectures
 
ECG CHALLENGE
ECG CHALLENGEECG CHALLENGE
ECG CHALLENGE
 
Pediatric Arrythmias
Pediatric ArrythmiasPediatric Arrythmias
Pediatric Arrythmias
 
Systematic ECG Interpretation
Systematic ECG InterpretationSystematic ECG Interpretation
Systematic ECG Interpretation
 
pediatrics.Cardiac arrhythmias.(dr.hader)
pediatrics.Cardiac arrhythmias.(dr.hader)pediatrics.Cardiac arrhythmias.(dr.hader)
pediatrics.Cardiac arrhythmias.(dr.hader)
 
Significance of avR lead
Significance of avR leadSignificance of avR lead
Significance of avR lead
 
Svt
SvtSvt
Svt
 
Approach to child with congenital heart disease
Approach to child with congenital heart diseaseApproach to child with congenital heart disease
Approach to child with congenital heart disease
 
OSCE Pediatrics KKCTH
OSCE Pediatrics KKCTHOSCE Pediatrics KKCTH
OSCE Pediatrics KKCTH
 
Approach to a Child with Congenital Heart Disese
Approach to a Child with Congenital Heart DiseseApproach to a Child with Congenital Heart Disese
Approach to a Child with Congenital Heart Disese
 
PALS - Pediatric advanced life support
PALS - Pediatric advanced life supportPALS - Pediatric advanced life support
PALS - Pediatric advanced life support
 
The ECG as a Diagnostic Tool: A Clinical Case Study of Supraventricular Tachy...
The ECG as a Diagnostic Tool: A Clinical Case Study of Supraventricular Tachy...The ECG as a Diagnostic Tool: A Clinical Case Study of Supraventricular Tachy...
The ECG as a Diagnostic Tool: A Clinical Case Study of Supraventricular Tachy...
 
Pediatric case presentation (congenital heart disease- PDA)
Pediatric case presentation (congenital heart disease- PDA)Pediatric case presentation (congenital heart disease- PDA)
Pediatric case presentation (congenital heart disease- PDA)
 

Viewers also liked (8)

Archer Dermatology USMLE step 3
Archer Dermatology USMLE step 3Archer Dermatology USMLE step 3
Archer Dermatology USMLE step 3
 
USMLE Step 2 CS Course
USMLE Step 2 CS Course USMLE Step 2 CS Course
USMLE Step 2 CS Course
 
Ethics in medicine - Archer USMLE step 3
Ethics in medicine - Archer USMLE step 3Ethics in medicine - Archer USMLE step 3
Ethics in medicine - Archer USMLE step 3
 
Acing the residency interview
Acing the residency interviewAcing the residency interview
Acing the residency interview
 
Usmle step 3
Usmle step 3Usmle step 3
Usmle step 3
 
25 most common diagnoses in 2011
25 most common diagnoses in 201125 most common diagnoses in 2011
25 most common diagnoses in 2011
 
Several Samples - Personal Statements
Several Samples - Personal StatementsSeveral Samples - Personal Statements
Several Samples - Personal Statements
 
Management of cbd stones a study
Management of cbd stones a studyManagement of cbd stones a study
Management of cbd stones a study
 

Recently uploaded

Radiation Dosimetry Parameters and Isodose Curves.pptx
Radiation Dosimetry Parameters and Isodose Curves.pptxRadiation Dosimetry Parameters and Isodose Curves.pptx
Radiation Dosimetry Parameters and Isodose Curves.pptxDr. Dheeraj Kumar
 
Primary headache and facial pain. (2024)
Primary headache and facial pain. (2024)Primary headache and facial pain. (2024)
Primary headache and facial pain. (2024)Mohamed Rizk Khodair
 
PNEUMOTHORAX AND ITS MANAGEMENTS.pdf
PNEUMOTHORAX   AND  ITS  MANAGEMENTS.pdfPNEUMOTHORAX   AND  ITS  MANAGEMENTS.pdf
PNEUMOTHORAX AND ITS MANAGEMENTS.pdfDolisha Warbi
 
Music Therapy's Impact in Palliative Care| IAPCON2024| Dr. Tara Rajendran
Music Therapy's Impact in Palliative Care| IAPCON2024| Dr. Tara RajendranMusic Therapy's Impact in Palliative Care| IAPCON2024| Dr. Tara Rajendran
Music Therapy's Impact in Palliative Care| IAPCON2024| Dr. Tara RajendranTara Rajendran
 
Study on the Impact of FOCUS-PDCA Management Model on the Disinfection Qualit...
Study on the Impact of FOCUS-PDCA Management Model on the Disinfection Qualit...Study on the Impact of FOCUS-PDCA Management Model on the Disinfection Qualit...
Study on the Impact of FOCUS-PDCA Management Model on the Disinfection Qualit...MehranMouzam
 
SGK HÓA SINH NĂNG LƯỢNG SINH HỌC 2006.pdf
SGK HÓA SINH NĂNG LƯỢNG SINH HỌC 2006.pdfSGK HÓA SINH NĂNG LƯỢNG SINH HỌC 2006.pdf
SGK HÓA SINH NĂNG LƯỢNG SINH HỌC 2006.pdfHongBiThi1
 
SWD (Short wave diathermy)- Physiotherapy.ppt
SWD (Short wave diathermy)- Physiotherapy.pptSWD (Short wave diathermy)- Physiotherapy.ppt
SWD (Short wave diathermy)- Physiotherapy.pptMumux Mirani
 
97111 47426 Call Girls In Delhi MUNIRKAA
97111 47426 Call Girls In Delhi MUNIRKAA97111 47426 Call Girls In Delhi MUNIRKAA
97111 47426 Call Girls In Delhi MUNIRKAAjennyeacort
 
PULMONARY EMBOLISM AND ITS MANAGEMENTS.pdf
PULMONARY EMBOLISM AND ITS MANAGEMENTS.pdfPULMONARY EMBOLISM AND ITS MANAGEMENTS.pdf
PULMONARY EMBOLISM AND ITS MANAGEMENTS.pdfDolisha Warbi
 
Apiculture Chapter 1. Introduction 2.ppt
Apiculture Chapter 1. Introduction 2.pptApiculture Chapter 1. Introduction 2.ppt
Apiculture Chapter 1. Introduction 2.pptkedirjemalharun
 
The next social challenge to public health: the information environment.pptx
The next social challenge to public health:  the information environment.pptxThe next social challenge to public health:  the information environment.pptx
The next social challenge to public health: the information environment.pptxTina Purnat
 
Informed Consent Empowering Healthcare Decision-Making.pptx
Informed Consent Empowering Healthcare Decision-Making.pptxInformed Consent Empowering Healthcare Decision-Making.pptx
Informed Consent Empowering Healthcare Decision-Making.pptxSasikiranMarri
 
Hematology and Immunology - Leukocytes Functions
Hematology and Immunology - Leukocytes FunctionsHematology and Immunology - Leukocytes Functions
Hematology and Immunology - Leukocytes FunctionsMedicoseAcademics
 
Presentation on General Anesthetics pdf.
Presentation on General Anesthetics pdf.Presentation on General Anesthetics pdf.
Presentation on General Anesthetics pdf.Prerana Jadhav
 
low cost antibiotic cement nail for infected non union.pptx
low cost antibiotic cement nail for infected non union.pptxlow cost antibiotic cement nail for infected non union.pptx
low cost antibiotic cement nail for infected non union.pptxdrashraf369
 
Tans femoral Amputee : Prosthetics Knee Joints.pptx
Tans femoral Amputee : Prosthetics Knee Joints.pptxTans femoral Amputee : Prosthetics Knee Joints.pptx
Tans femoral Amputee : Prosthetics Knee Joints.pptxKezaiah S
 
Case Report Peripartum Cardiomyopathy.pptx
Case Report Peripartum Cardiomyopathy.pptxCase Report Peripartum Cardiomyopathy.pptx
Case Report Peripartum Cardiomyopathy.pptxNiranjan Chavan
 
Glomerular Filtration and determinants of glomerular filtration .pptx
Glomerular Filtration and  determinants of glomerular filtration .pptxGlomerular Filtration and  determinants of glomerular filtration .pptx
Glomerular Filtration and determinants of glomerular filtration .pptxDr.Nusrat Tariq
 
Nutrition of OCD for my Nutritional Neuroscience Class
Nutrition of OCD for my Nutritional Neuroscience ClassNutrition of OCD for my Nutritional Neuroscience Class
Nutrition of OCD for my Nutritional Neuroscience Classmanuelazg2001
 

Recently uploaded (20)

Radiation Dosimetry Parameters and Isodose Curves.pptx
Radiation Dosimetry Parameters and Isodose Curves.pptxRadiation Dosimetry Parameters and Isodose Curves.pptx
Radiation Dosimetry Parameters and Isodose Curves.pptx
 
Primary headache and facial pain. (2024)
Primary headache and facial pain. (2024)Primary headache and facial pain. (2024)
Primary headache and facial pain. (2024)
 
PNEUMOTHORAX AND ITS MANAGEMENTS.pdf
PNEUMOTHORAX   AND  ITS  MANAGEMENTS.pdfPNEUMOTHORAX   AND  ITS  MANAGEMENTS.pdf
PNEUMOTHORAX AND ITS MANAGEMENTS.pdf
 
Music Therapy's Impact in Palliative Care| IAPCON2024| Dr. Tara Rajendran
Music Therapy's Impact in Palliative Care| IAPCON2024| Dr. Tara RajendranMusic Therapy's Impact in Palliative Care| IAPCON2024| Dr. Tara Rajendran
Music Therapy's Impact in Palliative Care| IAPCON2024| Dr. Tara Rajendran
 
Study on the Impact of FOCUS-PDCA Management Model on the Disinfection Qualit...
Study on the Impact of FOCUS-PDCA Management Model on the Disinfection Qualit...Study on the Impact of FOCUS-PDCA Management Model on the Disinfection Qualit...
Study on the Impact of FOCUS-PDCA Management Model on the Disinfection Qualit...
 
SGK HÓA SINH NĂNG LƯỢNG SINH HỌC 2006.pdf
SGK HÓA SINH NĂNG LƯỢNG SINH HỌC 2006.pdfSGK HÓA SINH NĂNG LƯỢNG SINH HỌC 2006.pdf
SGK HÓA SINH NĂNG LƯỢNG SINH HỌC 2006.pdf
 
SWD (Short wave diathermy)- Physiotherapy.ppt
SWD (Short wave diathermy)- Physiotherapy.pptSWD (Short wave diathermy)- Physiotherapy.ppt
SWD (Short wave diathermy)- Physiotherapy.ppt
 
97111 47426 Call Girls In Delhi MUNIRKAA
97111 47426 Call Girls In Delhi MUNIRKAA97111 47426 Call Girls In Delhi MUNIRKAA
97111 47426 Call Girls In Delhi MUNIRKAA
 
PULMONARY EMBOLISM AND ITS MANAGEMENTS.pdf
PULMONARY EMBOLISM AND ITS MANAGEMENTS.pdfPULMONARY EMBOLISM AND ITS MANAGEMENTS.pdf
PULMONARY EMBOLISM AND ITS MANAGEMENTS.pdf
 
Apiculture Chapter 1. Introduction 2.ppt
Apiculture Chapter 1. Introduction 2.pptApiculture Chapter 1. Introduction 2.ppt
Apiculture Chapter 1. Introduction 2.ppt
 
The next social challenge to public health: the information environment.pptx
The next social challenge to public health:  the information environment.pptxThe next social challenge to public health:  the information environment.pptx
The next social challenge to public health: the information environment.pptx
 
Informed Consent Empowering Healthcare Decision-Making.pptx
Informed Consent Empowering Healthcare Decision-Making.pptxInformed Consent Empowering Healthcare Decision-Making.pptx
Informed Consent Empowering Healthcare Decision-Making.pptx
 
Hematology and Immunology - Leukocytes Functions
Hematology and Immunology - Leukocytes FunctionsHematology and Immunology - Leukocytes Functions
Hematology and Immunology - Leukocytes Functions
 
Presentation on General Anesthetics pdf.
Presentation on General Anesthetics pdf.Presentation on General Anesthetics pdf.
Presentation on General Anesthetics pdf.
 
Epilepsy
EpilepsyEpilepsy
Epilepsy
 
low cost antibiotic cement nail for infected non union.pptx
low cost antibiotic cement nail for infected non union.pptxlow cost antibiotic cement nail for infected non union.pptx
low cost antibiotic cement nail for infected non union.pptx
 
Tans femoral Amputee : Prosthetics Knee Joints.pptx
Tans femoral Amputee : Prosthetics Knee Joints.pptxTans femoral Amputee : Prosthetics Knee Joints.pptx
Tans femoral Amputee : Prosthetics Knee Joints.pptx
 
Case Report Peripartum Cardiomyopathy.pptx
Case Report Peripartum Cardiomyopathy.pptxCase Report Peripartum Cardiomyopathy.pptx
Case Report Peripartum Cardiomyopathy.pptx
 
Glomerular Filtration and determinants of glomerular filtration .pptx
Glomerular Filtration and  determinants of glomerular filtration .pptxGlomerular Filtration and  determinants of glomerular filtration .pptx
Glomerular Filtration and determinants of glomerular filtration .pptx
 
Nutrition of OCD for my Nutritional Neuroscience Class
Nutrition of OCD for my Nutritional Neuroscience ClassNutrition of OCD for my Nutritional Neuroscience Class
Nutrition of OCD for my Nutritional Neuroscience Class
 

50 star ccs cases usmle forums

  • 1. 50 star ccs cases - USMLE Forums http://www.usmle-forums.com/usmle-step-3-recommended-thre... USMLE Forums Your Reliable USMLE Online Community 28,025 Members 98,912 Posts Home USMLE Articles USMLE News USMLE Polls USMLE Books USMLE Mobile Monthly Contest User Name User Name Remember Me? USMLE Forums > USMLE Step 3 Forum > USMLE Step 3 Recommended Threads 50 star ccs cases Password Log in Register FAQs Community Today's Posts Search Hi, Unregistered You have limited access as a guest. Join our community to have full access. Registration is fast, simple and absolutely free. Get your Free USMLE Consult™ Step 3 Question bank subscription when you post more in USMLE Step 3 Forum! Click Here for more details. USMLE Step 3 Recommended Threads Highly Recommended and Useful USMLE Step 3 Forum Threads. Thread Tools Search this Thread Display Modes 09-23-2009 #1 Steps History: 1+CK+CS Posts: 3 navz Threads: 1 USMLE Forums Newbie Thanked 10 Times in 1 Post Reputation: 20 50 star ccs cases found this on another forum so thought of sharing here . femur neck#--ER 2. Advanced maternal age 3. Snake bite 4. HEPATIC ENCEPHALOPATHY 5. Post-op atelectasis. 6. Septic Abortion- 7. Incomplete abortion with heavy bleeding and acute PID. 8. Infected peripheral IV access 9. atrophic vaginitis 10. polycystic ovarian syndrome 11. gono 12. AGN 13. BPH 14. acute psychotic disorder 15. spontaneous abortion 1 of 43 4/16/12 11:27 AM
  • 2. 50 star ccs cases - USMLE Forums http://www.usmle-forums.com/usmle-step-3-recommended-thre... 16. malrotation 17. child with rash 18. obesity in adolescent 19. CTS 20. teenager comes for a pre-employment 21. JRA 22. kawasaki disease 23. Osteomyelitis 24. Hypoglycemia 25. Dilated cardiomyopathy 26. Down's Syndrome neonate 27. Preeclampsia 28. MM 29. Hypercalcemia-office 30. Hypercalcemia-ER 31. MS 32. HELLP 33. Endometrial Hyperplasia without Atypia 34. Gastric cancer 35. TURP-HYPONATREMIA 36. testicular pain-acute unilateral, child—ER 37. hypertensive crisis with SAH 38. Hypertension-secondary 39. fever unknown origin – adult 40. septic shock 41. Alcohol withdrawal 42. retained placenta 43. Chronic renal failure 44. Acute renal failure 45. RAPE 46. HUS 47. New onset DM-42 yr old c polydipsia & polyuria 48. Fever Unknown origin-child 49. Cx ca 2 of 43 4/16/12 11:27 AM
  • 3. 50 star ccs cases - USMLE Forums http://www.usmle-forums.com/usmle-step-3-recommended-thre... 50. Failure to Thrive 1-femur neck#--ER Hx- 55 female post meno Exter. Rot. & shorten Right LL VS- stable P/E general skin heent look for hematoma heart lung abd cns extr Orders IVA oxy sat Pulse oxy cardiac monitor bp monitor EKG nss cbc bmp ua pt ptt inr fobt blood type,crossmatch Ca,phoph,pth,mg x ray –chest,hip(ap/lat) ,knee morphin-pain consent form orthopedic consult ; reason hip fracture admit to wd interval+rest ex cefazolin IV on time npo bed rest ,bathroom urine output foley ranitidine cbc/d bmp/d h&H ptt-4h heparin sq pneumatic surgery done- post-sx morphi ambulate early calcium vit d if shows patient improves, discharged and f.u after 2wks Counseling mobilization 3 of 43 4/16/12 11:27 AM
  • 4. 50 star ccs cases - USMLE Forums http://www.usmle-forums.com/usmle-step-3-recommended-thre... exercise alendronate medicine comp stop smoking limit alcohol seat belt calcium rich diet Dexa scan colonoscopy lipid mammography --------------------------------- 2-Advanced maternal age office: PE--complete. urine HCG (+ve) CBC BMP UA urine culture LFTs lipid profile blood type & cross type and Rh pt/ptt EKG pap smare USG trans vaginal mammography gonorrhea chlymydia RPR Hep B antibody HIV ELISA toxo rubella titer Educate patient pregnancy pregnant mother counseling no smoking, safe sex, seat belt, safety plan diet (high calory,High protein) iron oral folic acid oral multi vitamins-prenatal follow up after 1 month in 1st trimester 2nd visit full physical vital(BP check) Weight UA Fetal heart monitoring fundal height do triple screen --serum HCG, AFP, estradiol [MS AFP], if high amniocentesis Triple marker screen-TRIPLE TEST Valproic acid level send her home.. counsel about the vit and folic and RATED SEX further diagnostic plan continue Valproic acid Genetic counseling 4 of 43 4/16/12 11:27 AM
  • 5. 50 star ccs cases - USMLE Forums http://www.usmle-forums.com/usmle-step-3-recommended-thre... Genetic Sonogram Amniocentesis Karotyping of the fetal amniocytes....cells found in the amniotic fluid --------------------------------- 3-Snake bite ER Location 25 y young guy – 30min after hx of snake bite. Haven’t brought snake, order- vitals/hr IV Access NS Pulse oxi O2 Abg Cont BP moniter Cardiac moniter cbc bmp ua pt ptt Bleeding time ECG CXR Blood type and cross match. Foley Urine output NPO PE-(complete) shows local cellulitis, noticed 2 fang sites on his ankle Neuro xam shows drowsiness order- Polyvalent snake antivenom -SNAKE Shift to ICU interval hx+rest Ex Bedrest Vitalsq2h Pulse Oximetry q2h Neuro exam q2h Monitor Bleeding time, PT, PTT, Platelet counts (Can develop DIC) H&H-6h Ranitidine Inj TT Ampicillin/Cloxacillin IV ABG q8h cbc/24 Bmp/24 If Neuroparalyis symptoms---- (Atropine + Neostigmine IV) If devloping resp failure---- intubate and mech ventilation D/C ASV when Bleeding time/PT/PTT parameters normal and neuro symptoms subside, D/c IV antibiotics; make oral When pt ok—to wd Later send home counselling ----------------- 4-HEPATIC ENCEPHALOPATHY pt presented with altered state of mind... had h/o of cirrhosis of liver already. was given oxycodone my dentist following which he devleloped symptoms..HEP C cirrhosis LOOK FOR-alkalosisi,low k,SBP,HIGH PROT diet,….. ER routine (IV access, pulse oximetry, cont bp, cont cardiac moniter) 5 of 43 4/16/12 11:27 AM
  • 6. 50 star ccs cases - USMLE Forums http://www.usmle-forums.com/usmle-step-3-recommended-thre... fingers stick glu Thiamine inj 50% dxt NGT suction (to look for variceal bleeding) EKG CXR CBC, BMP, S.NH3 NPO NSS GPE( signs of cirrhosis, confused patient, asterixis) LFT, PT, PTT, urine r/e, toxicology, blood alcohol level ,Mg URINE culture BLOOD culture (pt marginally raised, lft deranged as for cirrhosis serum NH3 sky high) admit ICU urine output, bed rest, continue thiamine and dextrose normal saline drip, propranolol to control portal HTN./ spironolactone PARACENTESIS-SEND FLUID LEVOFLOXACIN----IF PARACENT+VE{>250CELL) CHANGE TO CEFTRIAXONE .Lactulose oral [ampicillin po or neosporin po/ neomycin] enemas to evacuate stool Moniter PT/PTT/ cbc/bmp/ammo daily . Pt improves –ADD diet salt restriction .shift to wards in 24 hrs or when better Case usually ends after 6 hrs of pt getting better. rehab 5-Post-op atelectasis. [after 36 hr of Surgery]... DD's were: Pneumothorax PE Pneumonia CXR--it showed Atelactasis It is one of the first cause of Fever, High WBC and shortness of breath... So.. I did..blood culture Removed the Foley Catheter and put a new one UA culture And Started.. IV Antibiotics Acetaminophen Per Rectal And Incentive Spirometry... Patient become stable...I transferred him to Inpatient Unit.. ---------------- 6-Septic Abortion- 6 of 43 4/16/12 11:27 AM
  • 7. 50 star ccs cases - USMLE Forums http://www.usmle-forums.com/usmle-step-3-recommended-thre... Do cultures IV Antibiotics Call OBGYN When patient is accepeted by OBGYN for D&E Just do medical Management in ICU 7-Incomplete abortion with heavy bleeding and acute PID. Do cultures IV Antibiotics Call OBGYN When patient is accepeted by OBGYN for D&E Just do medical Management in ICU -------------- 8-Infected peripheral IV access iva (if central line, dc cental line and new central line) oxy vitals q1h (qday when stable temp) cardiac monitor (risk of septic shock) fingerstick stat b-hcg cbc stat bmp stat focused pe pt/ptt stat ua/uc+s blood cx cxr esr crp xray site doppler arm remove iv line cath tip for c+s, gm stain, fungal cx clinda + zosyn (if admitted >48 hrs) tylenol ekg 2 d echo full pe elevate arm bed rest iv nss iv heparin if signs of cord-like thickening/bluish discoloration (new iv access already done in beginning to give empiric abx) when cx back: iv naf for 2-3 days (until improvement) dc zosyn and clinda in this case do not worry abt dc'ing patient or po meds and patient has iv line for a reason. 5 min: repeat cbc, chem in 3-4 days counsel screen ---------------- 9-atrophic vaginitis 62 yo vaginal itching clear discharge painful intercourse 7 of 43 4/16/12 11:27 AM
  • 8. 50 star ccs cases - USMLE Forums http://www.usmle-forums.com/usmle-step-3-recommended-thre... vulvar erythema mucus bleed during exam dd atrophic vag bacterial vagi candidial vagi vulvar ca cervicitis tricho office work up cbc cmp lipid profile as a health maintainance exam-safer to do bu no credit wet prep trich gono chly pap may need emb colpo etc if finding in cervix or pap if pap positive story goes further otherwise cou vaginal gelly for lubrication local hrt estrogen cream follow up as needed ----------------- 10-polycystic ovarian syndrome 21 yo f beard excessive hair weight gain menstrual irregularity darkening axillry thickened skin normal vitals dd polycystic ovarian syndrome congenital late onset adrenal hyperplasia adrenal tumour drug effects like minoxidil phenytoin ovarian neoplasm cushing syndrome idiopathic hirsutism cbc lft bmp endocrine--dhea lh fsh ratio prolactin tsh dhea 8 of 43 4/16/12 11:27 AM
  • 9. 50 star ccs cases - USMLE Forums http://www.usmle-forums.com/usmle-step-3-recommended-thre... testerone blood glucose insulin level serum 17 hydroxyprogesterone us pelvis result - testerone increse lh fsh ratio increase insulin fasting glucose ratio increse urine pregnancy test -do it anyway -- rx ocp exercise metformin spirolactone smoking cesation fu 6 month ---------------- 11-gono-male 21 m unprotected sex urethral discharge fever sickness burning sensation during urination o/e urethral discharge red urethra suprapubic tender d/d -acute cystitis epidymitis forign body nephrolithiasis orchitis prostitis pyelonephritis reiter's syndrome urethritis --gono chlymydia office w/u ua culture urethral gram stain urethral discharge for gono chlymydia vdrl cbc finding -- gram stain gram neg culture awited rx azithromycin 1 gram stat 9 of 43 4/16/12 11:27 AM
  • 10. 50 star ccs cases - USMLE Forums http://www.usmle-forums.com/usmle-step-3-recommended-thre... ceftrixone 250 mg stat coun safe sex practice smoking alcohol safe driving drug culture -- fu 4 week pt coun treat patner ------------- 12-AGN 10 m tea urine priorbit edema had fever with hx of sore thrat 3 wk bak bp 140/85 ankle edema dx -cryoglobunemia iga nephropathy membranoprliferative gn post streo gn er work cbc chem 8 ua no need of cs he does not have fever 24 hour urine protein aso titer complement -low ua--proteinuria wbc cast rbc cast rx lasix captropril penicillin office work up us renal throat culture office rx furosemide captropril nephrology consult - fu 3 week family couns dietary consult low sodium diet fluid restriction seat belt ---------- 13-BPH 70m sono need of pregnancy test? 10 of 43 4/16/12 11:27 AM
  • 11. 50 star ccs cases - USMLE Forums http://www.usmle-forums.com/usmle-step-3-recommended-thre... night urin frequency urgency hesitency terminal dribbling double micturation weak stream sensation of incomplete evacuation vita wnl prostate normal but enlarged office cbc bmp-urea creatinine normakl ua cs us prostate psa esr residual urinary volume rx finesteride prazocin which is a selecting short acting alpha blocker second visit urology consult urodynamic study fu six month for dre and psa dietary consult seat belt smoking alcohol patient counseling ----------- 14-acute psychotic disorder dd mania bipolar 1 stress malingering panic scizophrenia drug delirium vital s -wnl so pe meds- olazapine valproic acid we should give a antipsychotic and mood sabliser- lithium or valproic acid order-cbc bmp no need of lft pt ptt order tsh uds no need of ua no need of ekg xray cardiac enzymes do psychotherpy psychiatry consult coun med compliance 11 of 43 4/16/12 11:27 AM
  • 12. 50 star ccs cases - USMLE Forums http://www.usmle-forums.com/usmle-step-3-recommended-thre... suicide contract regular exercise patient education send home appointment - one week agin exam if not allright -vdrl hiv sle ect can be given monitor cbc with antipsychotic ------------- 15-spontaneous abortion 27 yo f lmp 6 week ago lower abdominal crmp vag bleed cervix - open blood in vault vitals tachy bp wnl dd ectopic abortion polyp cancer inflammation or cervicitis normal menstration with dysmenorrhoea er work up cbc pregnancy test qualitative then quantiatative us blood group rh iv saline no cervix tenderness - no pap gono cz now hb -9 no bllod transfusion now pt is stable us -- fetus dead - fetal pole uterine pregnancy gyn consult for d and c d and c admit to ward iv saline pneumatic compression methylprednisolone doxy cbc folow up grief counselling counsel pt rebirth control follow up 3 week ------- 16-malrotation VOLVULUS 1 dy old m bilious vomi 12 of 43 4/16/12 11:27 AM
  • 13. 50 star ccs cases - USMLE Forums http://www.usmle-forums.com/usmle-step-3-recommended-thre... poor feed lethargy rectal bleed oe- distension 170 pulse 89 sat dd duodenal atresia intestinal atresia malrotation with volvulus meconium plug necrotising enterocolitis will do gi series to r/o duodenal will do plain xray will r/0 infections transfer to er iv aceess iv normal saline o2 abg cbc bmp lft abdominal xray cxr BLOOD C/S if fever abg-metabolic acidosis- means something in the intestine cbc leucocytosis- axr-airless rectum large gastric bubble- means some obstruction rx as intestinal obst rx--NPO ng tube suction iv bicarb if ph less than 7 pediatric surgery consult ward - upper gi sries - barium enema ng tube suction upper gi -- bird beak corkscrew proximal jejunum barium enema cecum in RUQ rx ng tube suction iv normal saline bmp fu 48 hours 13 of 43 4/16/12 11:27 AM
  • 14. 50 star ccs cases - USMLE Forums http://www.usmle-forums.com/usmle-step-3-recommended-thre... family counselling ---------------------- 17-child with rash ruleout drug reactin?? rule out lyme if suspected Office W/U Complete PE CBC, stat BMP, stat Pulse oximetry ESR, routine blood culture UA, urine culture CXR, stat EKG, 12 lead, stat Neck x ray, stat culture of scraping from rash No aspirin send the patient home and F/U in 4 days ------------- 18-obesity in adolescent Complete PE Measure Height Measure Weight Calculate BMI -- you have to do this manually...not in the CCS software cbc BMP LFT Fasting Plasma Glucose Fasting Lipid Profile Serum TSH UA 24 hour urinary cortisol If the age of the PT is 2-7 years old and BMI with 95 percentile......or more without complication, the goal should generally be maintenance of baseline weight, For children 2–7 years old with BMI at the 95th percentile or above and secondary complications, weight loss is indicated For children older than 7 years with BMI between the 85th and 95th percentile, without complications, weight maintenance is an appropriate goal. I guess this patient's BMI >95th percentile so, Weight Loss diet Counsel Patient for Exercise Program Counsel Patient Counsel Family Follow-up visit in 4 months If no change... Sibutramine or Orlistat, po Follow up in 6 weeks If morbid obesity, BMI more than 45 14 of 43 4/16/12 11:27 AM
  • 15. 50 star ccs cases - USMLE Forums http://www.usmle-forums.com/usmle-step-3-recommended-thre... Consider Bariatric surgery ------------- 19-CTS pe xray cbc esr crp tsh ra ana ca magnesium bmp lipid hcg fu 3 days night splint-SPLINT EXTREMITY nerve conduction study nsaid usual counselling another visit carpal tunnel confirmed another cou 3 month not better ortho consult for surg 20-teenager comes for a pre-employment she is 5ft 2 inches and weighs 180 lbs. Bp 155/90 pt eats fast food.. it is all about weight loss.... bmp show no Potasium drop abdominal exam ...no striae.... PE does not point towards high cortisol for her BP do urinanalysis chest x ray ekg bmp for her bmi lipid profile fasting FBS TSH cbc follow up one week bp in both arm counselling life style - low sodium low fat low cholesterol 15 of 43 4/16/12 11:27 AM
  • 16. 50 star ccs cases - USMLE Forums http://www.usmle-forums.com/usmle-step-3-recommended-thre... weight mange oreder calorie count and exercise folow up three month better not better add hctz it is not secondary hypertension bmp normal no cushing no coarctation ---------------- 21-JRA 7 yr old girl with fever, rash and polyarthritis PE: Complete Order: cbc ( stat ) bmp or may be just BUN and Creatinine (stat) ESR ANA RF UA LFTs Xrays of the joints involved PT/PTT ( if planning to do arthrocenthesis ) athrocenthesis can be done as well.... CULTURE-URINE/BLOOD/THROAT Tylenol CXR 12 Lead EKG Echo d-dimers and fibrinogen ( i do not know the indication ..maybe coz it's a vasculitis) will pretend the child does not have a high fever...so send him home. see in next 4-5 days with the lab results.. Results: Cxr of joint without erosive changes...( so no methorexate) ANA positive such pts get eye involvement, thus eye exam q 3mths RF positive BUN nad CR wnl ASPIRIN MTX-SECOND LINE Interval hx: improved Order Rheumatology consult Eye consult Physical therapy consult EXERCISE Med compliance Multi-Vits Calcium supplements/ diet rich in calcium Educate family MRI and Dexa--? then do RATED SEX...mneumonic counsellin...whatever is applicable 22-kawasaki disease..... < 5yrs of age fever, rash, conjunctival injection, cervical lympphadenitis, inflammation of lips and the oral cavity, redness and swelling of the hands and feet. coronary arteries aneurysm unknown etiology. 16 of 43 4/16/12 11:27 AM
  • 17. 50 star ccs cases - USMLE Forums http://www.usmle-forums.com/usmle-step-3-recommended-thre... PE ; complete Orders: pulse ox oxygen cbc bmp lfts esr ua 12 lead ekg bld cxs LP Urine cx CXR IVA results: thrombocytosis elevated ESR sterile pyuria EKG=ST seg depression and T-wave flattening mild hypoalbuminemia ORDER: 2-D echo Coronary angiogram....maybe??? Aspirin ( untill pt is afebrile for several days) IVIG Consult Peads Cardiologist (like Dr. Fisher says on CCS always Consult; it wont harm U) should continue 3-5mg/kg/day, d/c after 6-8 weeks if no signs of coronary involvement and practically indefinitely if there is a coronary problem. Influenza vaccine before starting aspirin to prevent REYE'S Syndrome MMR and Varicella to be delayed till 11 months INTERVAL HX: PT HAS DEFERVESCED I do not know what to do now....maybe... ORDER: d/c home on aspirin f/u in 7 days cbc on follow up may be in 30 days to look for platelets esr follow up repeat ECHO. 6-8 weeks out counselling... 23-Osteomyelitis PE; complete..except breast, genitalia and rectal * Orders: admit iv acess iv saline blood cs urine ua and cs cbc bmp pt ptt ESR C-reactive protiens 17 of 43 4/16/12 11:27 AM
  • 18. 50 star ccs cases - USMLE Forums http://www.usmle-forums.com/usmle-step-3-recommended-thre... X-ray of the involved leg ~Results...x-ray wnl... ESR 90..... *Order MRI or Bone scan( if MRI is... C/I)..........MRI more specific! Bone Biopsy ~Results ..... MRI= Mild destruction fo tibia... Bone Bx=GM shows neutrophils & Stph aureu grows on cxs! * Admit to ward diets bedrest with bathrm privilages cbc for day #2 Empiric coverage with Oxacillin & Cipro ...OR.... Ofloxacin & ceftriaxone....treatment for 6 -12 wks....IV... Gram negative osteomyelitis treated with Cipro orallay. * 5 minute screen RATED SEX age appropriate tests... ----------------- 24-Hypoglycemia 27 yrs old female nurse found unresponcsive, daiphoretic and tachycardic. Prior to this she wa sc/o headache and tremors. Pupils are wnl. PmHx is insignificant. diffrential includes Insulinoma Exogenous Insulin SU overdose Prolonged fasting O2 pulse ox IVA Vitals Q 1 HR BP Cards ekg CXR Accuecheck Beta Hcg urine thiamine dextrose 50 naltrexone- if pupil constricted then iv infusion-5%dex ~PE : HEENT, LUNGS, HEART AND ABDOMEN *Result BS 50 and pupils are wnl ~ORDERS: cbc bmp cal mag phos lfts UA abg C-peptide Insulin serum Insulin antibodies Bld alcohol level Urine tox 18 of 43 4/16/12 11:27 AM
  • 19. 50 star ccs cases - USMLE Forums http://www.usmle-forums.com/usmle-step-3-recommended-thre... SU urine screen TSH Cortisol level Lipid panel ~PE: come back and finish the exam now. * Interval histoyr...pt is a little awake ~Results: C-peptide rasied and SU urine tscreen +ve for glyburide! ~Order: Octreotide SC x 1 bolus do ct/ABD to see insulinoma gastroenterology surgeon consult for surgey * Interval Hx and VItals: improving ~Orders: Transfere to ICU Octreotide sc q 8hrs Accue check q 1hrs NPO Urine output teds bedrest cbc in AM bmp in AM Psychiatyr consult * Interval History/vitals check... * improved.. d/c npo, bedrest and octreotide and dextrose.when BS in the range of 85-90 * Move to the ward.... ~ "5 MINUTE SCREEN" PAP Rated SEX ..whatever is applicable screening (mamo if age >40) colono if age>50 counsel (I select as many counsellings for all patients as poss) diet consult suicide contract if OD resched visit in another 4 wks f/u in 2 weeks after the discharge... ================================================== ===== If insulinoma is suspected..then CT abdomen or USG abdomen.. DEBULKING i.e surgery is the treatment then..... anyone still feels the need to add something...lol...be my guest.. this is an exhausting one, for sure! 25-Dilated cardiomyopathy 55 yr old pt presents w bilateral LE edema, sob on exerction, no cough. h/o drinking 5 quarts of wine every day. PE bibasilar wet crackles and evidence of moderate ascites!!! Casuses of dialted CM alcohol adriaamycin radiation viral myocarditis amyloidosis 19 of 43 4/16/12 11:27 AM
  • 20. 50 star ccs cases - USMLE Forums http://www.usmle-forums.com/usmle-step-3-recommended-thre... sarcoidosis hemachromatosis Thiamine deficiency ~ Order: put thme in as STAT O2 Pulse OX EKG CXR Cards Vitals q 2 CBC BMP IVA Fingerstick PE ; COMPLETE Cal Mag Phos Lfts Amylase Lipase UA PT PTT lipid tsh e cho B12 FOLIC BLOOD ALCOH HEAD ELEV * Results : CXR=Enlarged heart w Kerley B llines + EKG= ST-T waves non-specific abnl ~ Admit to floor ~ ~Order IVA Low Na diet Bedrest w bath rm privilages TEDS Hepari SC Lasix IV KCL MORPHI Foley Strict Input and Output Daily weights Cardiac Echo ...now! CXR Q day BMP q day * interval hx...pt hope meds will help ~Result: echo shows dilated heart w EF 25% ~Order iv carvedilol iv spirono iv lisino iv digi Anticoagulation...consider in longterm .... if evidence of thrombosis strict daily wt, i/o (foley for strict uo) daily mvi, thiamine and folate (commonly deficient) bed rest low salt diet 20 of 43 4/16/12 11:27 AM
  • 21. 50 star ccs cases - USMLE Forums http://www.usmle-forums.com/usmle-step-3-recommended-thre... fluid restriction statin if abn lipids cards consult D/C ALCOHOL ACE receptor blockers for those who cannot tolerate ACEI when stable: dc all iv meds-->po—day3 f/u in 1 wk with another bmp and ekg echo in 2-3 wks screen counsel dietician consult ~ 5 Minute Screen alcohol anonymous alcohol abstienance lipid colonospcopy Rated SEX RPT ECHO-3MONTHS CARD REHAB Maybe you will have to manage the pt for 2-3 days in the simulated time...on the software... but d/c home on ace, beta bxs, spironolactone, dig and lasix...with follow up in next 7-14 days... Of course change IV meds to PO befor discharge ------------------------- 26-Down's Syndrome neonate new born downs most probably presents with vomitting ( duoenal atresia) telemetry Pulse oxy IVA iv one fourth normal saline oxygen BP monitoring Cardiac MONITOR NPO NGT IV metoclopramide stat Brief physical ABG CBC BMP CXR EKG AXR acute series USG abd UA urin culture LFTs amylase lipase USG confirms the Diagnosis Consult Pediatrics GI surgery Transfer to ICU 21 of 43 4/16/12 11:27 AM
  • 22. 50 star ccs cases - USMLE Forums http://www.usmle-forums.com/usmle-step-3-recommended-thre... vitals Q 1 hr NPO Urin out put karyotyping BP check electrolytes Karyotype confirms Down's ECHO audiometry TSH Genetics consult ------------------------------------- 27-Preeclampsia 21 yr 0ld at 33wks gestation, c/o facial and upperextremity edema.... how will you proceed pulse oxy IVA BP monitor Cardiac Monitor brief physical CBC BMP LFTs blood typr n cross match ( if not done already) pt/ptt UA Obtretical USG DEXAMETH MGSO4-IM[deli/labo] IF SEVERE PRE-ecl- Im-MgSO4 stat-cont IV hydralazine stat-cont ECL IV MgSO4 stat-cont IV hydralazine stat-con Urin creatinine ( it will be included in UA I guess) Transfer to ward interval hx-check-neuro pulse oxy Q 2hrs Vitals Q 2 hrs urin output bp check complete bed rest serum Mg Q 4 hrs-if give Urin 24 hr uric acid FHR monitoring fetal doppler OB consult observe for 24 hrs pt gets better deliver the baby after –term-nvd with oxy ------------------------ 22 of 43 4/16/12 11:27 AM
  • 23. 50 star ccs cases - USMLE Forums http://www.usmle-forums.com/usmle-step-3-recommended-thre... 28-MM Multiple Myeloma** patient presenting with Hypercalcemia CCS: hypecalcemia in a pt who is presenting acutely...... lets say a 45 year old male presents with abdominal pain, wife reports he has been acting a bit strange over the last few days.... how would you proceed.... Presented in emergency.! Differential will be following: 90% Primary Hyperparathyroidism Malignancy--- 1-* Osteolytic Hypercalcemia due to Myeloma, Lymphoma, Breast carcinoma 2-*Humoral Hypercalcemia-PtHrP – Sq Cell Cacinoma of lungs, head & neck, renal or bladder. 3-*Tumoral Calcitriol production is Hodgkins & Non Hodgkin Lymphomas. ~ORDERS: Iv Access Pulse oximetry Oxygen inhalation continuous Vitals x 1hr Focused PE : General, HEENT , Heart , Lung, Abdomen, Extremities ~ORDERS: cbc- ----- Stat bmp ------Stat calcium--- Stat Mag ---Stat Phos ---Stat Lfts ---Stat UA ---Stat EKG ---Stat CXR ---Stat Move the clock get the result ~*Results show Ca 13.5mg/dl ~*Results show Hb 8.7g/dl . Also BUN and Creatinine slightly Increased ~* EKG shows shortened QT Interval ~ORDER; Iv 0.9% Saline Continuous Salmon Calcitonin - SC q 6-12 hours Or IV Pamidronate Continuos (over 2-6 hours0 PTH assay 24 hr urinary calcium Sulfosalicylic Acid Test (to detect ~*Bence Jones Protein… coz Normal urine dipsticks will not detect light chain) Seum Protein Electrophoresis Urine Protein electrophoresis Seum alkaline phosphatase (to rule our Hyperparathyroidism & Paget disease) Serum Ferritin TIBC Serum Iron Ultrasound of Abdomen (to rule out Renal carcinoma) ~*Do Interval and Check for volume overload by focused Heart & lungs ~ORDER; If Volume overload then give Iv Furosemide One time only *******Patients feels better so move the patient to Ward ~ORDER; Vitals q 2 hours Iron Enriched Diet Ambulate at will Urine output BMP daily Sodium Docusate (stool softner) 23 of 43 4/16/12 11:27 AM
  • 24. 50 star ccs cases - USMLE Forums http://www.usmle-forums.com/usmle-step-3-recommended-thre... Calcium Daily Bone Xrays Move the clock … get results ~* Results show Bone X ray *Lytic Lesions. ~* Results show SPEP and UPEP *Positive ~* Results show PTH .*Normal” Order *Bone Marrow Biopsy Consent for Procedure Move the clock …Get the biopsy result ~* which shows “Plasma cells” in Bone Marrow ~*Call Oncology consult... Patient with Multiple Myeloma needs chemotherapy ~ORDER; Chemotherapy ---Vincristine, Adriamycin, Dexamaethasone. Hopefully case ends here 5 minute screen do all the counselling and age appropriate test. Epogen** SC to fix his anemia----- erythropoietin ------------------------- 29-Hypercalcemia-off ice 30 yr old with hyper calcemia on routine tests with h/o renal stones some 2-3 yrs bcak. Serum calcium is 11.5 mg/dl. PE complete except breast, recal and genitalia ~ORDERS: cbc bmp calcium Mag Phos Lfts PT PTT UA EKG CXR send the pt home..call him when all the lab result return! Pt returns...do a small PE Results show ca 11.5mmg/dl ORDER; PTH assay 24 hr urinary calcium Dexa Scan pt home, call with results of the tests Results show..PTH 23 mEq/ml Call general surgery consult...parathyroidectomy..... as pt fulfills 2 criteria for undergo parathyroidectomy + age less than 50 and h/o Renal stones.. Hopefully case ends here 5 minute screen do all the counselling 24 of 43 4/16/12 11:27 AM
  • 25. 50 star ccs cases - USMLE Forums http://www.usmle-forums.com/usmle-step-3-recommended-thre... and age appropriate test. ------------------------- 30-Hypercalcemia-ER a 45 year old male presents with abdominal pain, wife reports he has been ating a bit strange over the last few days.... iva pulse oxi stat vitals q1h cardiac monitor cxr abdo usg cbc stat bmp stat ca, mg, phos stat ua stat amylase stat lipase lfts tsh pt/ptt morphine iv full physical npo sr. pth 24 hr ur. ca spep/upep vit d level abdo ct normal saline iv furosemide iv calcitonin if better: tt ward vitals q4h bmp, ca, mg, phos q4h consult surg if pth for parathoidectomy (blood type and cross match, npo, foley) dc all iv's alendronate send home after 48 hr f/u in 1 wk with cbc, bmp, ca, mg, phos, sr. pth 5 min screen: colono counsel f/u in 4 wks with same labs ---------------------- 31-MS 24 yr F pt comes to your office with blurring of vision in L eye and weakness of right leg. Past history of such weakness episode on R arm prior to 6 months.--ER PE(complete) fundoscopy shows blurring of disc margin, disc atrophy; admit to ward 25 of 43 4/16/12 11:27 AM
  • 26. 50 star ccs cases - USMLE Forums http://www.usmle-forums.com/usmle-step-3-recommended-thre... Pregnancy test cbc bmp ua tsh vit b12 pt/ptt mri brain and spine( shows demylinating lesions) iv methyl prednisolone consult neurology consult opthal LP( shows oligoclonal bands) neuro check-2h baclofen for spasticity if pain gabapentin bladder hyperactivity oxybutinin fatigue amantadine or flouxetine urinary retention bethanecol Contracpetive normal diet/urine output/ambulation at will r/w after 12 hrs interval history and PE symptom free-- on day3 d/c IV methyl pred Fasting blood sugar cbc bmp calcium vitaminD Interferon or Glatimer acetate start counsel patient contraception oral predni vacc-influ eye consult follow up appointment; after 3 months MRI repeat ------------------------ 32-HELLP 35 wks getation...bp 170/115, headache, scotoma, epigastic pain-ER pulse ox Oxygen cardiac monitors Bp continous IV access NS NPO MgSO4..IV Latetalol..IV DEXAmethasone.. IM P/E Admit to ICU... 26 of 43 4/16/12 11:27 AM
  • 27. 50 star ccs cases - USMLE Forums http://www.usmle-forums.com/usmle-step-3-recommended-thre... BED U OUT PNEU RANITIDINE-IV CBC BMP Ca, Mg. Phos LFTs----------------CONSULT OB PT/PTT Retic FSPs D-Dimers UA USS-PELV Labor monitor FETAL MONITOR CULTURE-VAG,GONO,CHLAM MONITOR—BP,PLT,UA-PROTEIN,PT.PTT,BMP type and cross 2 units of PRBCs FFP Platelet PENICILLIN G-IV OXYTOCIN NVD 5MIN H&H ORAL DIET PT PTT OMEPRA BF PAP LFT ---------------------------------- 33-Endometrial Hyperplasia without Atypia hx if imp for age, if she wants childbirth or contraception. usually presents with abnormal heavy uterine bleed. (if very heavy: send to ER), lets assume here its moderate, no distress, pt now in office. full PE CBC BMP UA LFTS B-HCG TSH PT/PTT BLEEDING TIME PAP ENDO BX USG-PELVIS-ENDO THICKNESS Call her again in 2 days: if anemic, order FOBT, RETIC, PERI SMEAR, TIBC, SR. IRON, FERR RESULTS ALL WNL. BX SHOWS HYPERPLASIA WITHOUT ATYPIA CYCLIC- MONOPHASIC OCP'S IF SHE DESIRES CONTRACEPTION 27 of 43 4/16/12 11:27 AM
  • 28. 50 star ccs cases - USMLE Forums http://www.usmle-forums.com/usmle-step-3-recommended-thre... OR MEDROXYPROGESTERONE ORAL FOR 14 DAYS IF NOT FESO4 MVI Call back in 14 days IMPROVED BLEEDING: NO SYMPTOMS: CAN DC MEDROXY IF PERSISTENT: CONTINUE MEDROXY FOR 6 MONTHS Call in 1 month to check 5 min screen: pap x1 yr mammo endo bx x6months Colono counsel IF OLD PATIENT: SEVERE BLEEDING: HYSTERECTOMY -------- 34-Gastric cancer Weight loss , Abdominal pain , Nausea, presntation was Gastric obstruction Dysphagia, Melena, Early satiety ,Ulcer-type pain NPO NG Tube IV Fluid Vitals Iva Pulse oxy ABG CARDIAC MONITOR CBC BMP UA CXR EKG XRAY-ABD-series P/E FOBT LFT LIP ID FBS PT PTT ALBUMIN CA,MG,PHOS AMYLASE LIPASE PHENERGAN MORPHINE WD OUTPUT Barium upper GI studies CONSENT CONSULT-GASTRO ENDOSCOPY BIOPSY-NO OPTIONIN SOFTWARE 28 of 43 4/16/12 11:27 AM
  • 29. 50 star ccs cases - USMLE Forums http://www.usmle-forums.com/usmle-step-3-recommended-thre... COLONOSCOPY ENDOSCOPY,BX ADENO CA ANEMIA-IRON STUDIES,VITB12 FESO4, VITA C CT-CHEST CT-ABD PT PTT TYPE NEXT ORDER CONSENT CONSULT –GASTRO-SX CONSULT-RADIO CONSULT-ONCO CONSULT-DIETICIAN ADVANCE DIRECT -------------------------------------------- 35-TURP-HYPONATRE MIA Manifestations of the TURP Syndrome: • - acute hypo-osmolality • - acute hyponatremia • - congestive heart failure • - pulmonary edema • - hypertension • - hypotension • - solutee toxicity: hyperglycinaemia (glycine) hyperammonaemia (glycine if detected intra-operatively bleeding points should be coagulated, surgery terminated as soon as possible and iv.fluid should be stopped OXY VITALS CARDIO BP MONIT OXY FOLEY CULTURE-BLOOD/URINE CBC BMP UA EKG CXR DC-NSS DC-SX FUROSEMIDE-IV-AFTER LOW Na ICU BED NPO OUT PT PTT 29 of 43 4/16/12 11:27 AM
  • 30. 50 star ccs cases - USMLE Forums http://www.usmle-forums.com/usmle-step-3-recommended-thre... CBC HandH TYPE ABG PULSE OXY IF SEIZURE/confusion- DIAZEPAM 3% NACL ----------------------------- 36-testicular pain-acute unilateral, child--ER testicular torsion, - the most dramatic and potentially serious of the acute processes torsion of the appendix testis,-MC epididymitis. abrupt onset of severe testicular or scrotal pain.N,V awaken with scrotal pain in the middle of the night or in the morning VITALS-FEVER NO IVA OXY CARDIO BP MONITOR CBC BMP UA/culture PHENERGAN MORPHIN P/E- GENTAL,ABD�� �HORIZONT LEVEL CREMESTERIC REFLEX-NEG NPO TRANSILLUMINATION USS-SCROTUM--------TEST:TORSION CX R amylase lipase LFTs PT PTT TYPE CONSULT-PED SX Cefazolin B/L ORCHEOPEXY Orchiectomy is performed if the testicle is nonviable IF FEVER+POSITIVE CREMESTER WITH NEG USS---ACUTE EPIDIDYMITIS frequency, dysuria, urethral discharge –UTI CULTURE CHILD WTH UTI-CEPHALEXIN,BACTRIM GONO- CEFTR+DOXY scrotal support, rest ---------------------------------------------- 30 of 43 4/16/12 11:27 AM
  • 31. 50 star ccs cases - USMLE Forums http://www.usmle-forums.com/usmle-step-3-recommended-thre... 37-hype rtensive crisis with SAH Ischemic stroke or subarachnoid or intracerebral hemorrhage -- Intravenous labetalol Other first-line agents include transdermal nitroglycerin paste and intravenous nicardipine nitroprusside should be considered second-line therapy Sublingual nifedipine should be avoided o Antihypertensive agents previously were advocated for an SBP greater than 160 mm Hg or diastolic BP (DBP) greater than 90 mm Hg. o Keep systolic blood pressure 90-140 mm Hg before aneurysm treatment, then allow hypertension to keep systolic blood pressure less than 200 mm Hg Acute pulmonary edema -- Nitroprusside or nitroglycerin with a loop diuretic Drugs that increase cardiac work (hydralazine) or decrease cardiac contractility (labetalol or other beta blocker) should be avoided Angina pectoris or acute myocardial infarction - nitroprusside and nitroglycerin Aortic dissection - beta blocker such as propranolol or labetalol. +/- Nitroprusside Noncontrast brain CT or brain MRI Electrocardiogram Complete blood count including platelets Cardiac enzymes and troponin Electrolytes, urea nitrogen, creatinine Serum glucose Prothrombin time and international normalized ratio (INR) Partial thromboplastin time Oxygen saturation Lipid profile Lumbar puncture if subarachnoid hemorrhage is suspected and head CT scan is negative for blood Electroencephalogram if seizures are suspected ------------------------------------ 38-Hypertension-secondary youn g man no risk factor cbc bmp lft pt ptt inr lipid tsh ua uds cxr ekg FBS home if sodium high potassium lo normal anion gap give KCL office go for aldesterone/rennin activity ratio if high-24 aldosterone level spiranolactone abd/ct 31 of 43 4/16/12 11:27 AM
  • 32. 50 star ccs cases - USMLE Forums http://www.usmle-forums.com/usmle-step-3-recommended-thre... monitor BMp home ct-adrenal mass ward consent consult pt ptt type npo laparoscopic adrenalectomy -------------------------------- if cxr show cardimeg rib notching-MRA go for coarctation if ua proteinuria do ultrasound kidney mara kidney do nephro consult do surgey consult measure bp in both arm start meds beata-2 hctz-1 acei ccb smoking obesity alcohol drug hx coumsel call back -------------------------- 39-fever unknown origin - adult h pe cbc bmp lft pt ptt inr cxr ekg ua uds if prtinent blood culture 32 of 43 4/16/12 11:27 AM
  • 33. 50 star ccs cases - USMLE Forums http://www.usmle-forums.com/usmle-step-3-recommended-thre... urine cs sputum lp if meningitis suspected or alter mental ct if necessary gyn consult -- if gyn cause surgery if abscess if lft increse hepatitis panel if central lines line culture if janeway osler roth spot or bacterial endo--echo if throat pain lad mono atypical lypho in blood rapid strep test if viral syndrome vdrl hiv if leg pain or sob or pe suspected vq scan later -doppler or low -d dimer if no improve - joint lymph node ry eye dry mouth connective tissue panel treatwith abx iv saline npo if sepsis suspected surgical consult and ct if abscess see the bllod cs report change the abx no response you may have to add amphotericin if herpes thing - add acyclovir ----------------------------------------- 40-septic shock 33 of 43 4/16/12 11:27 AM
  • 34. 50 star ccs cases - USMLE Forums http://www.usmle-forums.com/usmle-step-3-recommended-thre... fever shock tachy low bp left lowe quadrant pain iv normal saline iv access oxygen pulse oxy later abg focus pe cbc lft pt ptt ua uds urine culture blood culture times two ct abdomen and pelvis with contreast lactic acid cxr echo if bacterial endocarditis suspected--later in floor if he does not improve can do cardiac enzyme to rule out cardio shock amylase lipase xray abdomen - do or dont do because yo do ct anyway start abx cefotaxime genta intake out put foley urine out put 1 hourly pt get bette -continue ct reort comes surgical consult for drainage of abscess better dc with cipro ------------------------------- 41-Alcohol withdrawal tachy,sweating,tremor,agitated iva o2 pULSE OX cARDS MONITORS bP Accue check PE... real quick HEENT RESP CARDS Labs... STAT.. npo nss cbc bmp lfts 34 of 43 4/16/12 11:27 AM
  • 35. 50 star ccs cases - USMLE Forums http://www.usmle-forums.com/usmle-step-3-recommended-thre... Blood alcohol level Urine toxicology ABG amylase lipase PT PTT Thiamine IV Folic acid Iv Calcium serum Mag srum Phos serum now come back and Complete the PE...what ever is lfet librium transfere to ICU seizure precaution aspiration precaution 5 minute screen counselling RATED SEX... alcohol anonymous.. ----------------------------------- 42-retained placenta (ER,3post op D, w fever and abd tenderness) Pulse Oxymetry IV Access IV NSS Complete PE CBC with Diff PT/PTT Blood Grouping and Cross Match LFTs UA and CS Blood Cultures Cervical Cultures and gram staining IV Ceftriaxzone IV Clindamycin Consult OBG, for retained placental removal send the pt to medical ward: Bed rest NPO Vitals Q4H ---------------------------------- 43-Chronic renal failure in office take complete physical exam 35 of 43 4/16/12 11:27 AM
  • 36. 50 star ccs cases - USMLE Forums http://www.usmle-forums.com/usmle-step-3-recommended-thre... order vitals cbc bmp ua cxt abd ultra sound abg ekg serum lipids serum albumin serum calcium phosphate vitamin D pth lft if pt in emergency then we do iva and also see the urnie output check phosphate lever and also pt ptt and do blood typing as tehre might be anaemia so we mite need transfusion or even dialysis if acidosis calcium tratment is diet ---------------------------------- 44-Acute renal failure 80 yr old man comes to ER with n/v and maliase. PMhx is significant for Htn, DM and osteoarthritis. Pt is on NSAIDS, lisinopril. Also reports making very little urine over the last 24 hrs. PE : General, heent, LUNGS, ABDOMEN ~ Orders: iva nss pulse ox vitals Q 2 hrs cards ABG Accue check ekg and cxr CBC BMP Cal Mag Phos UA Urine cxs LFTs tylenol ~PE: come back and complete the rest of the exam now... *!* Results..(.pH 7.29, PCO2 20, PO2 80). (BMP NA 138 & HCO3 12) 36 of 43 4/16/12 11:27 AM
  • 37. 50 star ccs cases - USMLE Forums http://www.usmle-forums.com/usmle-step-3-recommended-thre... ~Transfere to ward ~Order low potassium diet/diabetic diet bed rest with bathroom privilages d/c NSAIDS d/c Lisinopril FOLEY Strict input/putput Teds Urine NA ( NL IS LESS THAN & EQUAL TO 10) and Creatnine 24 hr Urine protien eosinnophils in urine...(seen in allergic nephritis) Renal USG ( if BPH...call urology consult) Hgb A1c DAILY WEIGHTS Accue check q 4 hrs Insulin sliding scale ( if need be ) BMP q 2-4 hrs hopefully pt starts to improve after d/c nsaids and NSS infusion... pre-renal RF treated with fluids...if no rsponse...IV lasix.... Dobutamine and dopamine (if heart failure) ~famous 5 minute screen RATED SEX...what ever is applicable. --------------------------------------------- 45-rape complete physical orders: maybe one ste of vitals.. RAPE KIT... cbc..for baseline UA pregnancy test..beta HCG urine cervical smear KOH prep Hanging drop cervical gram stain and culturegonorrhoe DNA probe testing Chlamydia DNA probe testing morning after pill..i think it is LEVONORGESTREL-oral(high dose estrogen) for 2days Now STD prophylaxis: Ceftriiaxone 125 mg IM Azithromax 1gm PO Probenecid Metonidazole 2gm po for trichomonas add vdrl rape crisis consult cervical sample for chlamy and gonorr elisa for HIV Hep B surface antigen Social services consult Psych Consult? ---------------- 46- HUS 37 of 43 4/16/12 11:27 AM
  • 38. 50 star ccs cases - USMLE Forums http://www.usmle-forums.com/usmle-step-3-recommended-thre... ER Focused PE IVA NSS Oxygen pulse oxymetry Monitor Blood Pressure NPO CBC BMP UA, Culture AXR acute series Stool leuco, cultu, ova & para peripheral smear ldh haptoglonin in/output pt/inr pt/ptt blood type cross match D-DIMER FIBRIN DEGRADATION PRODUCTS-fdp results come as low platelet fragmented RBC no FDP no D dimers PT/PTT are normal K+ is elevated treatment with keyexalate Once stable transfer to ICU monitor BP cbc check BMP again every 1 hr till K+ normalizes followed by every 4hrs pt/ptt supporitve for now. consult hema pediac counsel pat/fam If case doesn't improve plasmapheresis ***check for ldh inc. schisto in peripheral, retic increa BUN & crea are in BMP ---------------- 47-New onset DM-42 yr old c polydipsia & polyuria DD- DM, DI, Factitious Disorder since it is a clinic setting...no emergency.. Pe: complete ORDERS: cbc bmp ca, mg, phos UA 12ekg ABG lipid profile cxr HgbAIC lft 38 of 43 4/16/12 11:27 AM
  • 39. 50 star ccs cases - USMLE Forums http://www.usmle-forums.com/usmle-step-3-recommended-thre... Finger stick BS 325mg/dL.—DIAGNOSTIC DM Admit to inpx service monofilament skin test, serum/urine ketones, serum/urine osmolarity, urine microalbumin. UA-CULTURE -ivf nss, -sliding scale:bld glucose 100-200-do nothing bld glucose 200-300-5u insulin bld glucose 300-400-10u insulin blood glucose (accucheck or finger stick) q2h, BMP-2h -vitals q4h, Activity prn, foley's catheter for intake output,1800 ADA (55-60% cho,less tham 30% fat,15-20% protein,vitamins,minerals,H20), lisinopril and other anti HTN (if HTN) -podiatry or chiropody consult -endocrinilogy consult -ophthalmology consult -If insulin 100-200,dc insulin and institute glipizide, dc ivf -counsel:weight loss,diet,exercise, annual ophthalmologic exam, foot care and protection,medication compliance, depression couseling, family counseling,medication side effect counseling,annual health maintenance and flu vaccine couseling. -follow-ups. -------------------- 48-Fever Unknown origin-child INFANT-bac,HSV Child-infect,connective Cbc Ua Bmp Cu lture-blood,urine,throat Lft Ana Rf Esr Cxr Lp-irritable Ppd Hiv Syph Ct-abd Wbc scan-gallium/indium NO-emp..ABx --------------------------------------- ----------- 49-Cervical cancer 39 of 43 4/16/12 11:27 AM
  • 40. 50 star ccs cases - USMLE Forums http://www.usmle-forums.com/usmle-step-3-recommended-thre... physical exam cbc bmp ua urine beta hcg gonorhea probe chlamydia probe wet mount vaginal ph HIV ELISA VDRL pap move the clock forward if has come with chlamydial infection/ginorrhea treat that call in 3 days (pap result comes in 3 days) Colposcopy Endocervical curretage(its there in the software but asks for Gynecolgy consult) Gynecology consult(No Endocervical biopsy on the software so can ask for that also on the Gynec consult) Move the clock forward Call her in a week colpo-cx ca Interval history Admit to ward Bed rest with bathroom previleges cxr lft pelvic ex IVP CSYTOSCOPY SIGMOIDOSCOPY abdominal ct pelvic ct bone scan RADIO-CONS oncology consult ekg 12 leads blood type cross match pt ptt Serum Iron with TIBC reticulocyte count interval history TAH+BSO(If family done) Gynecology consult RADIATION/CHEMO-CISPLATIN patient education no smoking no alcohol supportive psychotherapy Iron enriched diet 50-Failure to Thrive 2yr - below 5th percent 40 of 43 4/16/12 11:27 AM
  • 41. 50 star ccs cases - USMLE Forums http://www.usmle-forums.com/usmle-step-3-recommended-thre... If severe malnutrition/abuse---Hospitalization dd- low intake abuse chd infection endo genetical Hx+EX Head,neck,weight Cbc Bmp Ua Cxr Fobt Lft Culture-stool/urine Folic Vit b12 Stool-ova/fat/ -- HIV PPD TSH Sweat test Galactose----- --- Caloric count Nutrient supp Cons-dietician Social service F/u-q week The Following 10 Users Say Thank You to navz For This Useful Post: angelina vivien (04-04-2011), chatti (09-02-2011), dr.dhruvdesai (06-16-2011), drrsahuja (11-12-2011), jahn77 (2 Weeks Ago), jatnpatl (05-18-2010), missmbbs (11-27-2010), mle_asap (05-05-2010), shaan (07-14-2010), tommylee (02-24-2012) Foods To Lower Cholesterol Healthy diets and tips. Find foods to lower cholesterol. BestHealthDiets.com Ask a Urologist Online A Urologist Will Assist You Now! Questions Answered Every 9 Seconds. Health.JustAnswer.com/Urology Test Your PT/INR at Home Check your own warfarin levels with Philips PT/INR Self Testing at home inrselftest.com 09-23-2009 #2 Steps History: Step 1 Only Posts: 5 carroline Threads: 0 USMLE Forums Newbie Thanked 1 Time in 1 Post Reputation: 11 woow I kept scrolling and scrolling down and this post never finish how reliable is this info! 41 of 43 4/16/12 11:27 AM
  • 42. 50 star ccs cases - USMLE Forums http://www.usmle-forums.com/usmle-step-3-recommended-thre... 09-23-2009 #3 Steps History: 1+CK+CS Posts: 3 navz Threads: 1 USMLE Forums Newbie Thanked 10 Times in 1 Post Reputation: 20 :) hahaha i think it is pretty short for 50 cases..bt i think it covers some vital stuff u that cud help u for sure while working wid uw stuff!!! 11-27-2010 #4 Steps History: 1+CK+CS Posts: 31 missmbbs Threads: 1 USMLE Forums Scout Thanked 9 Times in 8 Posts Reputation: 19 Thanks! thanks so much for the post! much appreciated. 09-02-2011 #5 Steps History: 1+CK+CS Posts: 1 chatti Threads: 0 USMLE Forums Newbie Thanked 0 Times in 0 Posts Reputation: 10 100 important USMLE STEP 3 CCS cases that's helpful this is a list of dr.red 100 important CCS cases , i think it will help http://www.slideshare.net/usmlegalaxy , in this see the dr.red pdf CCS cases 11-11-2011 #6 Steps History: 1+CK+CS+3 Posts: 64 slime66 Threads: 1 USMLE Forums Scout Thanked 3 Times in 3 Posts Reputation: 13 Thanks WOW!!! I am speechless. And Thanks. Tags Step-3-Preparation Similar Threads Thread Thread Starter Forum Replies Last Post 42 of 43 4/16/12 11:27 AM
  • 43. 50 star ccs cases - USMLE Forums http://www.usmle-forums.com/usmle-step-3-recommended-thre... Similar Threads Additional new cases in Third edition of FA frankly USMLE Step 2 CS Forum 8 11-14-2011 07:42 AM Step 2 CS complete and incomplete cases ath.pantelis USMLE Step 2 CS Forum 1 05-30-2010 07:34 AM Should we do UW Practice Cases! MikheilGM USMLE Step 2 CS Forum 4 01-17-2010 07:03 PM Pediatric cases! chikawawa USMLE Step 2 CS Forum 2 07-10-2009 09:10 PM Watch Full Episodes Turn Your Computer into a TV! Watch Full TV Episodes Online. www.TelevisionFanatic.com USMLE STEP 2 CS Review 5 Day LIVE Workshop To Prepare For Step 2 CS Exam... 99% PASS Rate! www.ximedus.com File for SSI Disability Find Out If You Qualify For Social Security Benefits. Free Evaluation! socialsecuritylawfirms.com/denied Home - Contact Us - Forum Rules - Privacy - Archive - Top USMLE® & other trade marks belong to their respective owners, read full disclaimer USMLE Forums created under Creative Commons 3.0 License. (2009-2012) 43 of 43 4/16/12 11:27 AM